Sei sulla pagina 1di 92
Ha Duy Hung Céc bai thi Olympic Todn quée té CAC BAI THI OLYMPIC TOAN QUOC TE ff THANH TICH CUA DOI TUYEN VIET NAM (Tirnam 1959 dén 2004) Phién 202072005 Copyright 2005 © By HaDuy Hung and www.diendantoanhoe.net Tai ligu nay téc gid nhugng lai cho ban quan tri cla w gitrbin quyén. Bat cif hinh thiie sao chép la ‘thuong mai déu phai c6 su déng ¥ clia ban quan tr} mang va Ha Duy Hung Céc bai thi Olympic Todn quée té ‘Toisé khong bao gids quén me toi boi chinh ngudi da gieo va nudi duong nhing hat gidng dau tien eita edi thign 6 trong toi. Ngudi da md long toi true nhimg cam nhan vé thien nhién; ngudi da dinb thie hiéu biet cia toi va m6 rong chan trdi cia toi; va nhiing ldi day bao cita ngudi C6 Anh hudng khong ngimg len ca cude di toi. Ha Duy Hung Céc bai thi Olympic Todn quée té Mot i quY ude khi doe 1. DHKHTN Ha Noi : Dai hoc Khoa hoc Tir nhién Ha Noi DHSP Ha Noi: Dai hoc Su pham Ha Noi THPT: Trung hoc Phé thong. v . IMO: International Mathematical Olympiad. HCV: Huy chuong Vang. HCB: Huy chuong Bac. HCD: Huy chuong Déng ged : Uée chung lén nat. a . lem: Boi chung bé nha. 10. AXY Z: Tam giée XYZ. 11. ab: achia hét cho b. 12. alba 1a wée cita b. 13. afb: Phan so $ 14. (a,5): Uée chung én nhat cia hai s6 a va b, 15, [x]: Phan nguyén cia sO thye «tie 18 sé nguyen 1én nhat khong vuot quar. 16, {27}: Phin 8 cia $6 thye ©, te 18 sO xe dinh bai ze — [ 17, per at tite WA ting 21 + ay beer tty 18. 9 cla n s6 nguyen duong diu tien, nghia 1+ 2++-n. 19, [[p ste te Fay + ny-+ 20. #, AB: Vector 7, vector AB. 21. [@|:Modul cia vector 7 22. Z, 2 Tap tat ca cdc s6 nguyen duong. 2 Tap tit ed cae s6 thu N: Taptat cd cde s6 nguyen khong am. 25. Q: Tap 26. (”) :Hé sé td hop. fc so hitu ty. Contents 1 Cude thi Olympic Toan qu 1.1K) thi Olympic Toan quée té. 1.2. Mot s6 thanh tich téng quan dang nhé cia doi tuyén Vigt Nam. « 1.3 Doi digu tan man . . 2 Cée bai thi Olympic Ton quéc té 2.1 Lan thit nhat, tai Bucharest, Romania, nam 1959... 6. eee eee ee 2.2. Léin thit hai, tai Bucharest, Romania, mim 1960. 0640 4seeeee ee 2.3. Lan thit ba, tai Hungary, nam 1961. . 2.4 Lain thit tu, tai Cong hoa Czechoslovakia, nam 1962 . . . 2.5. Lan thit nam, tai Wroclaw, Poland, nam 1963 2.6. Lain thit sdu, tai Moscow, USSR, nam 1964 . . . 27 Li 2.8 Lan thit tam, thi biy, tgi West Germany, nm 1965 6... ese e eee eee ee i Sophia, Bulgaria,nm 1966 2.2... eee eee ee 2.9 Lain thit chin, tai Cetinje, Yugoslavia nam 1967 . 2.11 Lan thit mudi mot, tai Romania, nam 1969... 2.12 Lan thit muti hai, 2.15 Léin thit mudi nam, tai Moscow, U.SS.R,nam 1973... 2.16 Léin thit mudi sdu, tai Erfurt, West Germany, nam 1974. 2.17 Lan thit mudi bay, tai Burgas, Bulgaria, nam 1975... . 2.18 Lan thit mudi tam, tai Linz, Austria,nim 1976 6... ee ee 2.19 Léin thit mudi chin, tai Belgrade, Yugoslavia,nam 1977.2... eee. 4 nim 1978.22... 2. 2.21 Lan thit hai mui mot, tai London, United Kingdom, nam 1979. 6... + 2.22 Lain thit hai muoi hai, tai Washington D.C, USA, nam 1981 2.23 Lain thit hai muoi ba, tai Budapest, Hungary, nam 1982 2.24 Lan thit hai mui tu, tai Paris, France, nam 1983 2.25 Lan thit hai muoi nam, tai Prague, Czet 2.26 Lan thit hai muoi sa 2.27 Ln thit hai muoi bély, 2.28 Lan thi hai muoi tam, tai Havana, Cuba, nam 1987. . 2.29 Léin thit hai muoi chin, tai Canberra, Austr: taiJoutsa, Finland, nam 1985 2... 66... ee i Warsaw, Poland,nam 1986 ......----- ia, nam 1988 2.30 Lan thit ba muoi, tai Brunswick, West Germany, nim 1989 we . People's Republic of China, nam 1990 2.32 Lan thit ba mui hai, tai Sigtuna, Sweden,ndm 1991... 2.6. eee 2.31 Lan thit ba muoi mot, tai Bei 2.33 Lain thit ba mui ba, tai Moscow, U: 2.34 Lan thit ba muoi tu, tai Istanbul, Turkey, nam 1993 2.35 Lan thit ba muoi nam, tai Hong Kong, China, nam 1994 . 2.36 Lan thit ba mui su, tai North York, Canada, nam 1995 . 2.37 Lan thit ba muoi biy, tai Mumbai 2.38 Lan thit ba muoi tém, tai Mar del Plata, Argentina, nim 1997 .. 2... 2.39 Lan thit ba muoi chin, tai Taipei, Taiwan, nim 1998 . . . Bucharest, Romania, nam 1999 . . 2.41 Lan thit bon muoi mét, tai Taejon, Korean, nam 2000. . 2.42 Lan thit b6n muci hi 2001 2.43 Lan thit bon muoi ba, tai Glasgow, Scoland, United Kingdom, nim 2002 . 2.44 Lan thif bon mui b6n, tai Tokyo, Japan,nam 2003 2... ee. eee ee 2.45 Lan thit bn muoi nam, tai Athens, Greece, nim 2004.4... 00-245 i Washington D.C., United States of Ameri 3. Lai gidi cdc bai thi Olympic Todn Quéc té 3.1 Lain thé bon muoi nam, tai Athens, Greece, nam 2004 . « 46 48 50 52 53 56 57 59 61 63 66 69 7 B 76 79 19 Chuong 1 Cuéc thi Olympic Toan quéc té 1.1 Ki thi Olympic Ton quéc té ‘Vaio nam 1959 theo sng kién cla Hoi Ton hoc Romania, ki thi Ton quéc té (IMO) chinh thite lin dau tin duge t6 chite tai thanh pho Bucharest cla Romania, Tren thye 1 ban du ki thi nay chi danh cho cic nude phe Xa hgi Chii nghifa, va Kin dau tign c6 bay nude tham gia Ia: Czechoslovakia (Tigp Khéc), Hungary, Poland (Balan), Romania, USSR (Lien XO), West Germany (Cong hod Dan chit Dic) va Yugoslavia (Nam Tu). Phai dgn sdu nam sau, te nam 1965 méi cé mot nude Phuong Tay dau tién tham gia ki thi ny va d IA Filand. Vigt Nam tham dy IMO Lin diu tién vio nam 1974, va c6 mot sit tring lap thi vi khi dé cting Ia nim ma United State of America (My) lin du tién tham gia IMO. Ngay trong lin di tign tham gia, nude ta da gay duge tigng vang lén tai ngay hoi Olympic todn hoc nay dé 1a doat mot huy chuong Vang, hai huy chuong Bac va mot huy chuong Déng. Thue té tren the gigi trong cdc nude khong tham du IMO lin dau tién vio nam 1959 06 I8 chi e Viét Nam va Trung Qué c6 duge thinh tich nay. Céch thie t6 chite ciia IMO ban diu 6 mots6 yu 16 chua én dinh, $6 bai todn thi trong cc ki thi IMO Lin thit nbat, nam 1959 16i nam 1962 thuing [a sau hode bay bai tosn. Chi bit du tir nam 1963 trédi s6 cde bai tosn trong mdi ki thi IMO la su bai duge déu cho hai ngay thi. Thang diém mdi bai todn thuéng dao dong tir 4 diém cho dén 7 digm va s6 diém t6i da tinh cho dén hét IMO nam 1979 1a 40 é thit hai muoi hai, nam 1981thi s6 diém t6i da mdi bai todn luon 1a 7 diém, va s6 digm t6i da 6 thé dat duge sau hai ngiy thi 1a 42 diém, Nam 1980 vi nhiing Ifdo chinh tri IMO di khong thé dién ra. Ban dau ban t6 chife chi cong bé gidi theo thé thite : Giai nhit, gidi hi, gidi ba va Khuycn khich. Vé sau nay dé ding véi tinh than Olympic da di thinh Huy chuong Vang, Bac, Déng, Bang khen hode gidy chimg nhan da tham gia IMO. Khi cudc thi IMO méi duge t6 chtic, mdi nude duge phép cit dén tam thi sinh tham du cude thi, Nam 1982 mi nude chi duge eit dén bon thanh vien, nhung vio nam 1983 thi 86 thinh vien mi dOi lai duge phép tang dén sdu va ké tit dé dén nay s6 thank vien 161 dama méi duge cit dén luon 1a s4u. Céc thf sinh tham dy IMO khong due vuot qué so tu6i 20 va nhimg ngudi dugc tham gia IMO bude phai I nhiing hoc sinh durge do tao & ‘trong nude (tite Ia néu nhur ban dang hoe phé thong & nuéie ngodi, ban sé khong duge dai 6 IMO: Céchai thi Olympic Todn quéc té 7 dign cho dat nut ciia minh dé tham gia IMO). Bén canh dé so lin tham dy IMO cing khong hé han ché, mién Ia anh dang hoc bac trung hoe phé thong, Nhiing ngudi tham dur IMO ciing khong nhat thiet phai dai dign cho mot dat nuée no ma anh ta hodn toan c6 thé gitidon vA tham dy tu do va diém s6 ciing sé duige tinh rieng biet, Dé co lé cing fa li do tai sao cho dén nay tong s6 diém mi doin c6 durge sau méi ki IMO lai khong dese thong ke chinh thic. Sau day 1a mot vai thong tin xung quanh céc ki thi IMO da dién ra 1959. IMO duoc té chite tai Bucharest cla Romania. Cé biy nude tham du. 1960. IMO duge té chite tai Bucharest ca Romania. Cé nim nude tham du. 1961. IMO duge de tai Hungary, Cé su nude tham du nam duy nhat y Grigorig Margulisda danh huy chong Bac cho doi tuyén U.S.S.R (Lien X0), ngudi sau nay a dinh dugc Giai thug Fields nam 1983. 1962. 1963. IMO duoc té chite tai Wroclaw, Poland. Cé tam nude tham dy. 1964, IMO duye té chite tai Moscow, U.S.S.R. Cé chin nude tham dy. 1965. IMO dutge t6 chife tai Berlin, East Germany. C6 muti nuéc tham du. 1966. IMO durge t6 chitc tai Sofia, Bulgaria. C6 chin nuée tham dy. Laszlo Lovasz va Jozsef Pelikan 18 hai thi sinh cing tham dy bon ki IMO va méi ngudi trong ho ‘mang vé cho doi tuyén Hungary ba huy chyong Vang va mét huy chyong Bac. 1967. IMO duge t6 chite tai Cetinje, Yugoslavia, C6 muti ba nude tham dy. Simon Norton doat huy chuong Vang cho di tuyén United Kingdom (U.K) ngay trong lin dau tham dy IMO, 1968. IMO duge 1969. IMO duge t6 che tai Bucharest ciia Romania, C6 musi bén nude tham dy. Simon Norton tri thanh ngudi du tien doat ba huy chuong Vang IMO cho doi tuyén U.K. Vladimir Drinfel'd dinh mot huy chuong Vang cho doi tuyén U.S.S.R, ngudi sau nay danh duge gidi thudng Fields nam 1990, 1970. IMO duge 1971. IMO due t6 chie tai Zilina, Czechoslovakia. Cé muti nim nuée tham dy. Day 1a in thet nim Wolfgang Burmeister tham dy IMO, Anh da danh cho doi tuyén East Germany ba huy chuong Vang va hai huy chuong Bac, 1972. IMO duge 1973. IMO duge hite tai Moscow, U. nuéc tham du. SR. Cécl nde tai Keszthely, Hungary, C6 mutsi bon nue tham du. hite tai Torun, Poland. Cé muti nam nude tham du. hitc tai Moscow, U.S.S.R. C6 muti su nude tham dy. IMO: Céchai thi Olympic Todn quéc té 8 tai Berlin, East Germany. C6 muti séu nude tham du. Da nam ma Jean-Christof Yoccoz danh duge huy chong Vang, ngudi sau nay danh due gidi thuéng Fields nam 1994. Day fa nam ma doi tuyén Viet Nam va U.S.A Jan dau tien tham dy IMO. 1975. IMO duge t6 chite tai Burgas, Bulgaria. C6 muti bay nude tham du. 1976. IMO duge té chite tai Linz, Austria. C6 mudi chin nude tham dy. 1977. IMO due t6 chite tai Belgrade, Yugoslavia, C6 hai mui nuée tham dy, Day 1a nam ma Richard Boreherds danh duge huy chuong Bac cho doi tuyén U.K, ngudi a danh duge gidi thudng Fields nam 1998. Peter Shor dng duge huy chuong Bac cho doi tuyén USA va anh dnh duge gidi thudng Nevanlinna nam 1998. 1978. IMO duge t6 chite tai Bucharest, Romania, C6 mutbi bly nutéc tham du. 1979, IMO duge 16 chite tai London, U.K. Cé hai mui ba nuéc tham dy. Alexander Razborov dinh due huy chuomg Vang cho di tuyén U.S.S.R. Anh danh duge gid thudng Nevanlinna nam 1990. 1980. Khong t6 chite thi IMO. 1981. IMO duoc té chite tai Washington, DC, USA. Cé hai muoi béy nude tham du. Timothy Gowers danh dugc huy chwong Vang cho doi tuyén U.K.,va danh duoc gidi thudng Fields nim 1998. Day [8 nam dau tien ma Canada tham due IMO. 1982. IMO dug t6 chit tai Budapest, Hungary. C6 ba muoi nude tham dy. 1983. IMO duoc t6 chiic tai Paris, France. C6 ba mui hai nude tham dy. 1984. IMO dug t6 chite tai Prague, Czechoslovakia. C6 ba muoi tu nude tham du. 1985, IMO duve t6 chit tai Joutsa, Finland. C6 ba muoi tim nude tham dy. 1986. IMO dug té chit tai Warsaw, Poland. Cé ba muoi bély nude tham du. 1987. IMO duye t6 chite tai Havana, Cuba. C6 bén muoi hai nude tham du. 1988. 1989, hfe tai Brunswick, Germany. C6 nam muoi hai nude tham dir, Neo 1a thi sinh dau tién dnb hai huy chong Vang cho d9i tuyén Vigt Nam. in th nam Martin Harterich tham dy IMO, anh da danh duge ba buy chuong Vang, mot huy chuong Bac va mot huy chuong Déng cho doi tuyén West Germany. Serge} Ivanov dnh duge chiée huy chung Vang thit ba ciia minh cho doi tuyén USSR 1990. 1991, 1992. 1993, 1994, 1995. 1996. 1997, IMO: Céchai thi Olympic Todn quéc té 9 IMO duoc t6 chite tai Beijing, China. C6 nam muoi tu nude tham dy. Andrew McMurry 18 thi sinh dautien danh huy chyong cho doi tuyén Ireland. IMO due 16 chite tai Sigtuna, Sweden. C6 nam mui nam nude tham du. Theodor Banica danh chiée huy chong Vang thi ba ciia minh cho doi tuyén Romania. Evgenija Malinnikova dinh duge chige huy chuong Vang thif ba ciia minh cho doi tuyén U.SS.R. Chi [8 thi sinh nit duy nat cho dén nay danh ba huy chuong Vang IMO. IMO due té chite tai Moscow, U.S.S.R. Cé nam muoi séu nude tham dy. Trinidad and Tobago va South Africa lan dau tién tham dy IMO. IMO due 16 chite tai Istanbul, Turkey. C6 bay mui ba nude tham dye, Macedonia lin dau tien tham dy IMO véi doi tuyén chi gém bén ngudi ty nhién da danh duge ba huy chuong Bac, Moldovia lin diu tién tham dir IMO. IMO duge t6 chic tai Hong Kong. Cé séu mui chin nude tham dy. Tai IMO Kin nay da c6 30 huy chuong Vang, 63 huy chuong Bac, 102 huy chuong Déng duge trao, Day 1A nim dau tign trong lich str tén tai etia IMO tt c& cae thanh vien ciia mot 0i tuyén danh duges6 diém 161 da, 42 diém. Doi tuyén da lap len ki tich nay 1a di tuyén U.S.A véi céc thanh vien cia d0i 18 Jeremy Bem, Aleksandr Khazanov, Jacob Shazeer, Stephen Wang, va Jonathan Weinstein. Oleg Alexandrov kin jén dinh huy chuong cho doi tuyén Moldavian (HCB). Nasko Karamanov 12 ngui dau tién dinh huy chuong Bac cho doi tuyén Macedonian IMO duge 16 chite tai North York, Canada, C6 bly muoi ba nude tham dy. Day 1a nam ma c6 s6 céc thf sinh dogt huy chuong Vang véi s6 diém 161 da nhiéu nhat: Nikolay Nikolov (Bulgaria), Song Liu (China), Cheng Chang (China), Chen- chang Zhu (China), Mihaly Barasz. (Hungary), Peter Burcsi (Hungary), Egmont Koblinger (Hungary), Maryam Mirzakhani (Iran), Sug-woo Shin (South Korea), Dragos Oprea (Romania), Ciprian Manolesco (Romania), Ovi nia), Serguei Norine (Russia), and Ngo Die Tuan (Vigt Nam), Hungary 18 ba dat nude e6 ba thi sinh dai dign doat s6 diém t6i da, Nikolai Nikolov Hin thi ba doat huy chuong Vang cho doi tuyén Bulgaria, Lukito Muliadi 1a thi sinh du tign doat huy churong cho doi tuyén Indonesia. IMO dutge t6 chic tai Mumbai India, C6 bly mui nam nude tham dy. Ciprian Manolesco (Romania) [a thi sinh duy nit dat so diém 161 da tai IMO Kin nay. Sachiko Nakajima 18 thi sinh nit dau tign dai dign cho d0i tuyén Japan tham dy IMO, ‘Yulij Samikov nh duge chiée huy chuong Vang thi ba cho doi tuyén Ukraine. IMO due t6 chite tai Mar del Plata, Argentina, C6 tim muti hai nuée tham du. Da c6 39 huy chwtong Vang, 70 huy chuong Bac, va 122 huy chung Déng duive trao, Ki thi nay ¢6 bon thi sinh doat s6 diém toi da la Carl Bosley (USA), Do Quoc Anh (Vietnam), Eaman Eftekhari (Iran), va Ciprian Manolescu (Romania). Ciprian Manolescu lin thit ba danh huy chong Vang cho doi tuyén Romania, Bruce Merry 12 thi sinh dau tign cia South African va thi sinh thit hai ciia African danh duge huy 1998. 1999, 2000. 2001 2002. 2003, IMO: Céchai thi Olympic Todn quéc té chuong Vang. Anh da tham gia IMO nam Lin va bén Lin danh duge huy chung, Sachiko Nakajima 18 nit thi sinh dau tien dai dién cho doi tuyén Japan dinh huy chuong Vang. IMO due t6 chic tai Taipei, Taiwan. C6 bly mui s4u nutée tham du, Ba c6 37 huy chung Vang, 67 huy chuong Bac va 102 huy chuong Déng duge trao, Omid Amini (Iran) 18 thi sinh duy nhat doat s6 diém ti da tai IMO Kin nay. Melanie ‘Wood 1a nit thi sinh di tign ciia U.S.A tham dy IMO va danh duge buy chong Bac. Nikolai Dourov lin thit ba danh huy chuong Vang cho doi tuyén Russia. Ivan Ivanov Lin thi ba danh huy chuong Vang cho doi tuyén Bulgaria, William Ted Blackwell IT danh huy chuong Bac va tré thanh thi sinh diiu tién cla Trinidad and Tobago danh due huy chuong, Oscar Felipe Bernal Pedraza tréthinh thi sinh dau tian dinh huy chuong Vang cho doi tuyén Colombia. IMO dupe t6 chite tai Bucharest, Romania, C6 tam mutoi mét nude tham dy. Ba 6 38 huy chuong Vang, 70 huy chuong Bac va 118 huy chuong Déng duge trao. Romania tré thanh dat nuée diu tién nam Lin dang cai IMO. Céc thi sinh doat sé digm cao nht tai IMO lan nay 12 Tamés Terpai (Hungary), Stefan Laurentiu Homet (Romania), va Maksym Fedorchuk (Ukraine) cing duge 39 diém. Tamés Terpai iin thit ba dnh huy chuomg Vang cho doi tuyén Hungary. Geoffrey Chu tré thinh, thisinh dau tign doat huy chuong Vang cho doi tuyén Australian, Raja Mukherji td thanh thi sinh du tign dnb hai huy chuong Vang cho doi tuyén Ireland, IMO duge 16 chite tai Taejon, Korea, C6 tém muti hai nude tham dur, Ba c6 39 huy chuong Vang. 71 huy chang Bac va 119 huy chong Déng duge trao. Tat ei inh cla doi tuyén China dnh duage huy chung Vang dé la Zhiwei Yun, iu, Xin Li, Qihui Zhu, Xinyi Yuan, va Zhongtao Wu. Nhimg thi sinh doat s6 digm t6i da 18: Alexandr Usnich (Belarus), Zhiwei Yun (China), Alexei Poiarkov (Russia), vi Alexandre Gaifoulline (Russia), Kentaro Ni anh huy chuong Vang cho doi tuyén Japan. Vladimir Dremov danh chiée huy chuong Vang thif ba cho dgi tuyén Russia, Reid Barton danh ba huy chuong Vang cho doituyén USA IMO due té chic tai Washington, DC, USA. C6 tm mug ba nuée tham dy. IMO duoc té chitc tai Glasgow, Scotland, UK. Cé tam muni tu nude tham dy. IMO duge t6 chic tai Tokyo, Japan, C6 tém mui hai nude tham du va hai nude agi quan sat vien v6i tng s6 457 thé sinh tham dy, Da c6 35 huy chung Vang 66 huy chong Bac va 100 huy chuong Déng duge trao, Day 1a nam ma dodn Vigt Nam dan duge hai huy chuong Vang véi s6 diém t6i da. }. IMO duoc té chite tai Athens Greece. C6 tém mui nam nude tham du. Lain dau tien Vict Nam danh duve bén huy chuong Vang tai mot ki ciia IMO. 10 di IMO: Céchai thi Olympic Todn quéc té én 1.2. Mot sé thanh tich tong quan dang nhé cua doi tu Viét Nam Sau day 1A mét vai thong tin c4 nhan dang nhé cia doi tuyén Viet Nam tai thi IMO ¢ Hoang Lé Minh, hoc sinh ciia trumg DHKHTN Ha Ne tien doat huy chuong Vang tai mot ki thi cia IMO. Ja hoc sinh Viet Nam dau ¢ Phan Vii Diém Hang, hoc sinh cia trung DHKHTN Ha Noi 1a hoc sinh nit Viet ‘Nam dau tién tham gia IMO (IMO 1975 tai Bulgaria). ¢ Le Ba Khanh Trinh, hoc sinh trang Quoc hoe Hué, 18 hoe sinh Vigt Nam dau tién doat huy chung Vang véi s6 diém t6i da, Anh ciing 1a ngudi duy nhit ciia Viet Nam 6 duge edi dip tai mot ki IMO: Huy chuong Vang IMO, va gi dic bit danh cho ngudi die dio, ¢ Nguyén Héng Thai, hoc sinh trudng THPT Chu Van An 1 hoe sinh Vigt Nam dau tien duge trao gidi hoe sinh tré tudi nhat tham du IMO (Anh doat huy chuong Déng nam 1978) ¢ Le Tu Quée Thang 1a hoe sinh diu tin cia trudmg THPT Le Hong Phong, Thanh pho Hé Chi Minh doat huy chuong Vang IMO véi so diém ti da. ¢ Dim Thanh Son l& hoc sinh dau tién ciia trudng DHKHTN doat huy chuong Vang IMO véi s6 diém ti da. tham dy IMO (doat hai huy chuong Bac). @ Ngo Bao Chau, hoc sinh trutng DHKHTN Ha Noi 1& hoc sinh Viet Nam dau tign i lin doat huy chuong Vang IMO, Hién anh dang gitt ki luc 1a ngu’i Viet Nam c6 téng sé diém qua cac ki IMO tham du cao nhat 42+ 40 = 82 diém. truéng DHKHTN Ha No i cc ki IMO. Anh ¢ Dio Hai Long, hoe si ‘Nam thit hai 1g 86 digm tai hoe sinh gui 6 ic Tuan, hoc sinh tru’ng DHKHTN Ha Noi l& hoe sinh in doat huy chuong Ving tai cdc ki thi IMO. Anh la ngudi tai céc ki thi IMO cao thit ba v6i 42-37 = 79 diém, Nam thé ba t6ng s6 diém ¢ DS Quang Yen, hoc sinh trudng THPT Lam Son, 14 hoc sinh diiu tign cia tinh ul dj IMO: Céchai thi Olympic Todn quéc té 12 ¢ Vii Ngoc Minh [a hoc sinh dau tién cia truémg DHSP Ha Noi, va Ia hoc sinh Viet ‘Nam thit tu hai lin doat huy chuong Vang IMO (tai cc IMO2001 va IMO2002), ¢ Lé Hing Viet Bao Ia hoc sinh thit nam ciia Vigt Nam hai lin doat huy chuong, ‘Vang IMO (tai cic IMO 2003 va IMO2004). Nhimg thi sinh ciia doi tuyén Viet Nam da timg doat huy chuong Vang IMO v6i sé diém téi daz 1. Lé Bé Khanh Trinh, hoc sinh truéng Quéc hoc Hué tai IMO 1979. 2. Lé Ty Quéc Thang hoc sinh truéng THPT Lé Héng Phong, Hé Chi Minh, tai IMO. 1982. 3. Dam Thanh Son, hoc sinh truing DHKHTN Ha Noi, tai IMO 1984. , tai IMO 1988. 5. Dinh Tién Cudng, hoc sinh tru’ng DHSP Ha Noi, tai IMO 1989. 4. Ngo Bao Chau, hoc sinh truing DHKHTN Ha No 6. Ngo Diic Tusin, hoe sinh trutng DHKHTN Ha Noi, tai IMO 1995 7. DS Quée Anh, hoc sinh tru’ng DHKHTN Ha Noi, 8. LeHing ai IMO 1997. iét Bao, hoc sinh truing DHKHTN Ha Noi, tai IMO 2003. 9, Nguyén Trong Cinh, hoe sinh trang DHSP Ha Noi, tai IMO 2003. ‘Mot s6 thanh tich dac biét khae: 1. Trudng DHKHTN Ha NOi 1a trudng du tien c6 hai hoe sinh doat huy chuong Vang trong cling mot ki thi IMO. Dén nay trudng DHKHTN di lap lai duge thanh ti ny ba Kin tai IMO 1995, 1996 va 2004. Dac bigt nam 2000 DHKHTN Ha No con Ip mot cai hatrick cuc ki xudt se: ¢6 ba hoc sinh cing doat huy chuong Vang IMO. v DHSP Ha Ne thi IMO (tai IMO 2002). inh doat huy chuong Vang tai mot ki 3. Nam 1978 la nam dau tién Vit Nam cé tlt cd céic hoc sinh tham dy IMO déu doat ii chinh thie. 4, Nam 198918 nam dai tign Vigt Nam c6 hai hoc sinh doat huy chuong Vang IMO, 5, Nam 1999 1& nam déu tien Viet Nam xép hang thit ba, thit hang cao nha tit truée én nay cia ta tai IMO. 6. Nam 1999 Lin diiu tign Vigt Nam c6 ba hoe sinh ciing doat huy chong Vang tai IMO. é 13 IMO: Céchai thi Olympic Todn qu 7. Nam 2002 lin dau tign Vigt Nam e6 bon hoe sinh ciing doat huy chuong Vang tai IMO. 8. Nam 2003 lin du tien Viet Nam c6 hai hoe sinh doat huy chuong Vang véi cing s6 diém toi da. Trong cdc dé thi IMO duge tinh bly duéi day, nam nao 6 Vigt Nam tham dy ching 16 sé dua them céc thong tin vé thinh vién doi tuyén, digm s6, huy chong doat duge dé ban doc tham khdo them. ! 1.3 Doi diéu tan man Nhin vio nhiing thong tin tren day chiing ta c6 thé thay due chudi nhiig thinh cong vang doi clia do tuyén Olympic Tosin quée gia Viet Num qua ede ngiy hoi elia IMO. C6 rat nhiéu cau hoi nguyén nhan no Vigt Nam, Cé niéu y kién duge dua ra, trong dé tham chi o6 lin t0i da doc mot bai béo ca tung doin IMO cia Hong Kong nam 2003 cho ring ¢6 Ié 1a do Viet Nam duge thira huGng su cong tic gitip do vé Giso duc va Dio tgo tir trude day ciia Lien X6 cd, ‘mot dait nude c6 thinh tich rat xuat sic 6 IMO. Nhumg chiing ta hay nhé lai rang tir rat sém nutée ta da ¢6 he thong dao tao mai nhon, 6 dé cdc l6p chuyén Todn, danh cho cde hoc sinh c6 nang khiéu Todn dic bigt duge {4p trung boi dung nang cao cdc kién thite vé toan cho cdcem. Chinh gido duc mii nhon, fa tién dé co ban va 1a nguyén nhan chinh dain dén chudi nhimg thanh cong chéi loi cia doi tuyén Viét Nam tai céc ngay hoi cla IMO. Tuy mhign c6 thé khang dinh rang céc than tich thi IMO khong phan anh duge lugng day va hoc Toan Viet Nam, Digu ndy eiing fa chuyén duong nhien bai thue myc dich eiia phong trio Olympic Ton hoc 1A khuyén khich con ngudi ta hoc Todn chinh, bén canh d6 18 tao ra moi trudng giao lu gitfa hoc sinh céc nutée c6 cing sé thich. chit khong phai 18 dé phat hién va ton vinh nhiing anh ti Toan hoc. Hiéu don gin thé nay, anh tham gia mot mén thi Olympic nao dé: Toan hoc, Bong da, Maraton, ... , thi ciing da duge ngudi ta tuyén chon ti truéc, kha ning ciia anh duge chon c6 thé xuat sac, c6 thé khong nhung quan trong nhat 1a phai phi: hop nhat véi cud thi tuong tmgma anh sé tham gia. Khong thé nio mot van dng vien béng chay, khong biet mo té gi vé bong 4 lai chonanh ta di thi da béng, cing nhu mot vi Gidost Toan hoc xusitsic gidi thi ede baitodn IMO vGi hoe sinh e6 himg thi véi ki thinay. ét bai mot cd nhan, Chinh tai c6 ba vain dé sau day Tai igu nay vin dang trong qué trinh hodn than va né duge v Vi thé ma sé khong thé trinh duge nhimg sai s6t dang tie. Hi ma tai ligu nay c6 thé sé mac phai 1. Thanh tich ciia doi tuyén Viet Nam tai céc ki thi IMO; rit 06 thé cin c6 mot s6 thong tin chua chinh xéc 6 day va rat mong céc ban gitip do néu c6 thé. "Cae thong tin nay ben canh mot s6 tai ligu tham khéo khéec, tée gid e6 tham khao bai viet rat bd ich cita Thay gido TS. Nguyén Viet Hai dang trong [8], chi tiét xin moi xem [9] é ‘on so xu. 3. Cong vige bign soan Idi gidi cde bai thi & IMO héu nhurt0i chua thyc hign duge 21. Hign tai t0i da hon thinh song Idi gidi tat ed cic bai thi IMO nam 2004 va Sip hon thanh nam 2003, Cac nam cdn lai néu ban no c6 ¥ dinh gitip 46 thi ede ban lu ¥ 18 nén bién soan bang file Tex hodic bign soan bing céc file TXT (hoe Word) nhung ding ngon ngit Tex va font ting Viet ABC. Day IA phien bin thit hai cha ti ligu nay va duge hodn thanh vao ngdy 02/07/2005. Phién ban méi ciia fi ligu nay néu khong cé gi thay di sé dugc updated vao cudi thang bay ‘nam 2005, khoang mot tudn sau khi IMO 2005 dign ra. Moi déng g6p xin giti vé dia chi Email: pluricomplex.amath@yahoo.com Digu cui cing ma toi mudi ban 6 day, 18 néu ban thich IMO hay tham gia va néu that bai diéu dé cting gidng nhu vige ban khong thé an keo khi dang dau rang, Toi chwa timg duge biét dn huong vi IMO, nhung t0i thay thich va quan tam dén né cho dii da qua roi cai tuéi duge tham du ngay hoi may. Vige viet thi ligu nay khong nm ngoai nhing am huting néi tren. Toi hy vong qua day ede ban sé c6 due mot sum tap diy dii va chinh. xiic cdc bai todin thi & IMO qua dé c6 thé sir dung nhur mot cdim nang nh’im cai thign kha nang ciia minh hode don giin la dé giai tri. Bang ra nén 6 Idi gidi hon chinh cho cdc bai ton, nhung cong vige bién soan stu tim céc Idi gidi 1A rat mat thoi gian, cong site i h6i ti ligu phi mang tinh Khoa hgc va Gido duc cao chit khong thé Kim theo kiéu mi an lign nhu tinh trang viet sch hign nay, Tuy thé hy vong trong tuong lai, tac gi han duge sir cong tac tir phfa cae ban dé ching ta c6 thé thuc hign duge diéu trén, Cui cling néu cdc ban chi doc tai ligu nay trén may tinh, cdc ban nén cdi Adobe Acrobat 6.0 Professional hodc mugn han bai cae phién bin nay sé cho phép thi ligu trong duge sng siia va dep hon cdc phien bin truée d6.? i Tigu may duge soan thao bang MikTex 4 Chuong 2 Cac bai thi Olympic Toan quéc té 2.1 Lan thir nhat, tai Bucharest, Romania, nam 1959 1. Ching minh ring phan s6 ee 2In+4 Mn +3 18 t6i gidn v6i moi s6 tw nhien n. # 2. Vai gid tri thuc nao ciia x thi biéu thie sau v+ 2c =1+ e—V2r—1 thoa man (6 day ta higu ring mots6 thue Mfy duge can bac hai thi sé d6 phai khong am) 3. Cho cde s6 thye a, b, e. Xét phutong trinh bac hai doi véi acos? x +beosx-+-e=0 Sir dung cfc $6 a,b va c hy dung mot phuog tinh bac hai doi v6i cos 2 ma e6 12 nghiém véi phuong trinh ban diu. So sénh cdc phutong trinh theo cos. va cos 2r khi ma a =4,b=2vae=~—1 4. Dung mot tam gide vudng véi canh huyén ¢ sao cho trung tuyén vé xuéng canh huyén bang trung binh cOng cia hai canh ben ciia tam gide. 'Chii thich cia ngudi dich: Phan s6 hitu ty © trong dé a,b € 2 duoc goi A t6i gidn néu hai sO a.va.b khong c6 uéc chung nguyen done {én hon 1 nao. 15 IMO: Céchai thi Olympic Todn quéc té 16 5. Mot diém M tuy ¥ duge lay ben trong doan thang AB. MCD va MBEF duoc dung tren cing nita mat phang ba AB, v6i céc doan thing AM, MB Jam canh tuong tg. Cac duéng tron ngoai ti¢p cdc hinh vudng dé véi ca tm la P va Q giao nhau Gdiém M va déng thai tai mot diém NV, Goi N" 1a diém giao ca hai dung thing AF va BC. (a) Chimg minh ring ede diém N va N" tring nhau. (b) Chimg minh ring cdc dudng thin MN di qua mot diém thude vao si lua chon cia digm M. $ khong phu (©) Hay xéc dinh quy tich cia trung diém cia doan thing PQ khi ma M thay déi gitta hai diém A va B. 6. Hai mat phing () va (Q) cat nhau theo giao tuyén (p). Mot diém A duge lay tren (P) va mot diém C’ duge lay trén (Q) sao cho ching khong nim trén (p). Dung mothinh thoi ABCD voi AB song song vai C’D sao cho mot dudng tron 6 thé ndi tiépduge va véi céc dinh B va D nam trong cdc mat phang (P) va (Q), 2.2. Lan thit hai, tai Bucharest, Romania, nam 1960 1. Hay xc dinh tat cd cde so N’ 6 ba chit s6 sao cho V chia hét cho 11 va sé bang tng cia tat cd cdc binh phutong cia cae chit s6 ciia NV. v Véi nhiing gid tri thye no cita x sao cho bit 7 bang nhau. Ta kihi¢u a 1a s6 do cla géc tai dinh A cha tam giéc con nim & chinh gitfa, Chimg minh ring tga= ino AABC véid6 ddi cde duimg cao ting véi dinh A va B lah, vay ia trung tuyén xuat phat tir AIA ma. 4, Dungtam gid cho trude, d6 5, Xét hinh lap phuong ABCD.A'B'C'D! (6 d6 mat ABCD dinh huéng nbu 1 mat A'BIC'D' (a) Tim qui tich céc trungdigm cia doan thing XY 6 d6 X 1a mot digm bat ki mot diém bat ki trén B'D’ 6. 2.3 IMO: Céchai thi Olympic Todn quéc té 7 xiic dinh quy tich ede diém Z nim trén doan thing X a) véi ZY =2XZ. Cho mot hinh nén tron xoay ngoai tiép mot hinh edu (hinh céu may tig xtic v6i ca day cia nén). Mot inh tru trdn xoay noi tigp hinh céiu néi trén sao cho mot trong hai day ciia hinh tru nim trén day ciia hinh nén, Kf higu V; 18 thé tich cla nén va Vj Bi thé tich cia hinh tru (a) Ching minh ring Vj # Va. (b) Hay xdc dinh s6 & bé nhat ma Vi = AV). Trong tnutng hgp k: tim duge hay dung g6e 66 dinh géc 12 dinh nén va gée dé truong mot doan 1a duding kinh ciia day hinh nén. 2 Mot hinh thang can ABCD v6i 6 dai duémg cao la h. fe day c6 do dai cho truée 18 AB =a vaCD = b, (a) Trén true d6i xting cla hinh thang 46, hy xdc dinh tat ef ede diém P sao cho ZAPD = ZBPC = 90". (b) Hay xéc dinh diéu kign sao cho cde diém P nhur trén 1a t6n tai (ch ¥ hay bign lun tat ed ede kha nang c6 thé xity ra. Lan thit ba, tai Hungary, nam 1961 . Gidi he phuong trinh sau day 6.46 a va b 1A cae hing sO thyc. Hay xéc dinh digu kign cilia a va b dé nghi¢m (x,y, 2) cla he thod man x, y, 2 1A.cdc s6 thye duong phan bigt. . Cho a, b va ¢ 1a dé dai ba canh ciia mot tam gidc c6 dién tich 1a F. Chimg minh ring 6 46.718 mot s6 tye Mhién cho trude. Giai phuong trinh cos? = sin" « Xét tam gide AP;P2P; va mot diém P nim ben trong tam gide d6. Cac dudmg thing PP, PP; va PP, cit cdc canh doi dign tai cde diém twong img la Qy, Qo va Qs. Chiing minh rng trong ba s6 PP PP. PQ” PQ PQs 3c clin dung & day thyc chat chinb la g6e tao boi hai dudng sinh, ma hai dudng sinh nay ct day tai hai diem la hai du mut cba mot dung kinh & day nén. dj IMO: Céchai thi Olympic Todn quéc té 18, C6 it nhat mot s6 khong Ién hon hai v: it nhat mot so khong nhé hon hai. 5, Hay dung mot tam gide AABC biét ring AC =b, AB =cva AM 1a trung diém cia doan thing BC vaa< 90°. ‘Ching minh ring bai toan lun 6 ft nhat mot nghigm hinh khi va chi khi bt gS < ¢ Say Ching minh ring a; = ay = On. 4, Hay tim nghigm 2), 2», 15.:°4,.r5 clia he phuong trinh sau day yr, yrt2 Yrs yrs 6 d6y [4 mot tham s6 thy. 5. Chting minh ring 6. Nam sinh vien A, B,C, D, E tham gia vio mot cude thi. Cé ngudi da tien doan ring \0c thi theo thi ty la ABCDE. Tuy nhien diéu nay lai khong diing. Tren thyc té thi khong c6 thi sinh no két thite cude thi véi vi tri ‘mA ngudi dé da tién doin, va khong c6 hai thi sinh mio duge dy doan Ia sé két tic lien ti¢p nhau lai thanh sur that, Mot ngudi khac tién doin rang cdc thi sinh sé két thie theo thit ty la DAECB. Thue té thi Idi tién doan nay c6 doi chit t6t hon, Cé diing hai thi sinh da ket thiie theo thé ty cita Idi tign dodn thi hai, va hai cap roi nhau cc sinh vign duge tién doan la sé ket thi lien ti¢p tren thue t da xdy ra digu d6. Hay xéc dinh thir ty két thi cia cdc thi sinh tham du cugc tl 2.6 1 v 6. 2.7 . Trong mat pl IMO: Céchai thi Olympic Todn quéc té 20 Lan thit s4u, tai Moscow, USSR, nam 1964 (a) Hay xc dinh tat cd cdc s6 nguyen dong n saocho 2" — 1 chia hét cho 7. (b) Ching minh ring khong cé so nguyén duong n nao thod man 2" + 1 chia hét cho 7. Giasit ring @, b, 18 do dai ba canh cia mot tam gide. Ching minh ring @%(b+e—a) +P(c-+a—b) + 2(a+b—e) < 3abe ‘Mot during tron noi tip tam giée ABC véi do dai ba canh cita n6 18 a,b,c. Dung -4c tiép tuyén t6i dung trdn song song véi cdc canh cita tam giée. Méi dudng thing trong ching déu cat ra mot tam. tirtam gide AABC. Trong mdi tam. gide dé mot du’ng tron noi tip dugc dumg. Hay xéc dinh tng dign tich cia bon dung tron ngi tigp néi trén theo a, b,c Cémudi biy ngudi lién Lac voi nhau qua email, mdi ngudi déu viet thu cho nhimg ngudi cin lai, Trong cde bite thur cla ho c6 ba chii dé duye dua ra thao luan, Mi cap chi thao lugn v6i nhau ding mot vain dé. Ching minh ring c6 ft nhait ba ngudi maciing thio lun véi nhau vé mot vain dé. ig cho nam diém thod man ring khong cé hai dudng thing nao trong 86 cdc duéng thang néi cac diém trong ching lai song song, vudng géc, hoac tring nhau. Tirmdi digm, cdc dung vudng géc duge vé t6i tate cdc dung thang noi bon digm cin Iai, Hay xdc dinh sé én nhait cde giao diém ma cae dubng vudng 26¢ d6 6 thé tao ra, Trong tit dien ABCD, dinh D duoc noi v6i Do la trong tam cia tam gide AABC. Céc dudng thang song song véi D Dy dugc ké qua cae diém A, B va C’. Che dung thing dé cét mat phing (BCD), (CAD) va (ABD) tai cdc diém Aj, By va C) tuiong ting. Chimg minh rang thé tich cia ABCD bang mot phain ba thé tich ca A, B,C; Dp. Két qua con dting khong néu diém Dj due chon bat ki trong tam gide AABC 2. Lan thit bay, tai West Germany, nam 1965 Hay xac dinh tat cd ede gid tri trong doan 0 < x < 27 thoa man bat ding thite eos. < |VT+ sin 2x — VI—sin 2x} < v2. .Xét he phutong trinh sau day Ay21 + Ait +Ar3t3 = 0 dy) + O32 + dagt3 3121 + dg2t2 + A333 = 0 I ° 2.8 1 v IMO: Céchai thi Olympic Todn quéc té 21 v6i ‘Ac bign din tim 18.2) 79,03, Cée h@ s6 tho’ man diéu kign sau day: (a) 431, 029, d33 Ia ede 86 dong; (b) cdc hé s6 cdn lai 12 am; (©) trong méi phutong trinh, téng cita cdc he s6 1A duong. Ching minh ring he da cho chi cé nghigm duy nat 1a Cho ti dign ABCD véicdc canh 1a AB va CD 6 d@ dai lin Juot 18 a va b twong mg. Khoing au AB va CD lad va g6c gitta ching la w. Té dign ABCD duge chia thinh hai hinh chép bai mat phing (c), song song véi cc dudg thing AB va C’D. Ty s6 gitta khong céch etia e tit AB va CD Ia bing k. Hay tinh ty s6 cita thé tich cba hai hinh chép nhan duge. Hay xéc dinh tat cd bo bon s6 thye 79, :r5,.r4 thod main ring tng cia bat ki mot 86 no véi tich ca ba s6 cin Iai Ia bang 2. Xét tam giée A ABC véi géc JOB nhon, Qua mot diém M ¥ O cae duéng thing wudng gée duge ké tit OA va OB, chan cia ching la cae diém P vaGQ tuong ing, Diém giao clia céc dudmg cao ciia tam gide AOPQ Ia IT. Héi ring quy tich cla digm 77 néu nh M chay qua (a) canh AB? (b) phan trong cia tam gide AOAB. ‘Trong mat phing c6 n digm (n > 3) cho truéie. MGi cap ede diém duge ndi véi hau béi mot doan thing. Cho d Ia do dai I6n nhat ciia cae doan thing 46. Ching ta xc dinh mot dung kinh cla mot tp hgp céc dim chinh 1A doan thang véi do dai bing d. Ching minh ring s6 cae duémg kinh cia mot tap hgp n phan tit cho trudc cho trude I& Khong vwuot qué n. Lan thit tam, tai Sophia, Bulgaria, nam 1966 Trong mot cude thi Todin hoe, c6 ba bai todn duge dé xudt la A, B va C. Trong so nhiing ngudi tham du, ¢6 25 sinh vien gidi duoc ft nhdt mot bai ton. Tat cdc tht sinh khong gidi duge bai A va s6 cdc thi sinh gii duge bai B bing gap hai kin so duge bai C. S6 cde sinh vién ma chi thi nhiéu hon 1 ic th sinh gidi duge bai A va mot bai todn khdc. Tat cd cdc sinh vien ‘A. Hoi ring S0 véi s6 ‘ma giai duge chi m9t bai bang mot niza s6 khong khong lam duge bi 6 baomhiéu sinh vign gidi duge bai ton B Cho a,b, ¢ 1a d0 dai ba canh ciia mot tam giée cdn a, 8, 7, tuong ting R& ba gée d6i dign véi ba canh trén. Ching minh rang néu atb=te(atga +btg9), thi tam gidc dang xét lA tam gide can. é 22 IMO: Céchai thi Olympic Todn qu 3. Ching minh ring téng cita cdc khodng céch tir ede dinh cla mot tit dign déu dén ‘tam cha hinh céu ngoai ti€p tt di¢n bé hon tong cila cdc khoang cach tit mot diém bait ki trong khong gian dén cdc dinh cia né dien. 4. Ching minh rang véi moi s6 nguyen duong n va véi moi sO thye x # kr /2"(t= O,1,...51; & Ris nguyen tuy 9) 1 1 1 cotgr—cotg 2" in2x sin 4a sin 2 5. Giai he phuong trinh sau day lay — @9| 9 + lay — a3| 273 lay — | 21 + [aa — 05) 25 las — ay] x1 + |ag — | ry (ay — a4] £1 + [ay—ag|e9 + lay —aaglerg 6 6.0, a2, 43,0, 12 bOn S6 thye phan bigt 6. Trén cdc canh BC, CA va AB ciia tam giée A ABC My cdc diém tuy 91a BK, L va ‘M tuong tng. Chiing minh ring trong ba tam giic AAML, ABK M va ACLK €6 mot tam giée ma dign tich ca né khong én hon mot nita din tich eiia tam g AAB 2.9 Lan thit chin, tai Cetinje, Yugoslavia nam 1967 1. Cho hinh binh hanh 4 BCD véi dé dai céc canh AB =a, AL lvavéi 4BAD= a. Ching minh ring néu A.ABD Pa tam gidc nhon thi bon dudng tron ban kinh 1 véicéctim la A, B,C, D phi hinh binh hinh khi va chi khi a 1). Trong ngay thit nhat mot huy chutong va 1/7 cba sé huy chung cdn lai duge trao, Trong ngay thit hai, hai chiée da duge trao cling véi 1/7 so huy chong ccon Iai; va cet tigp tue nhur vay. Trong ngay thé n cé.n huy chuong cdn lai va duoe tao hét. Hoi ring c6 bao nhieu ngay va bao mhiéu chiéc huy chuomg da duge trao. 2.10 Lan tht mudi, tai Moscow, U.S.S.R, nam 1968 1. Ching minh ring chi c6 mot va chi mot tam gide c6 d6 dai I& cde so nguyen Tien tigp thod man c6 mot géc 66 s6 do géip doi mot g6c khic cia cling tam gide d6. v Hay xdc dinh tat ca cc sé nguyén duong n tho man téng cdc chit s6 cla né trong hé co s6 mudi bang r? — 102 — 22. 3. Xét he phuong trinh sau day ar} + bey +e ar} + bry +c it bt_-1 +6= fn ard + bir, +e fe trong dé céc bign 21, 02,--+ ,,,cdna, b,c fa cée $6 thye va thod mana #0. Dat A= (b=1)? dae. Ching minh ring (a) Neu A < 0 thihe vo nghigm. (b) Néu A = 0 thihe ¢6 ding mot nghiem. (©) Néu A > O thi hé c6 nhigu hon mot nghigm. 4. Ching minh ring trong mot ti dign bat Ki déu c6 it nhat mot dinh ma ba canh chia 16 c6 thé lap thinh ba canh cia mot tam gie. 5. Cho f ki mot him gid tri thye xc dinh tren tap tat cd cic s6 thute 2 thoa man c fle+o)= 4+ VI@= TTP v6imoi 7 ER (trong dé a 1a hing s6 thuc). (a) Chimg minh ring f 1A ham tun hoan (nghia 1a t6n tai mot sO 6 thod man F(e-+b) = f(x) v6i moi). 6. é 24 (b) Véi a = 1 cho mot vi du vé mot ham khdc hang s6 thod man cdc tinh chat tren, ‘Vi mai s6 nguyen duong n hay tinh gid tri téng, Stsel-be} ea a (Trong dé [2] duge kf higu 1a s6 nguyen 1én nhat khong lén hon x.) 2.11 Lan thi mudi mot, tai Romania, nam 1969 1 v 6. Ching minh ring ¢6 v6 han s6 nguyen duong a thod man sé n+ a khong pha l& 86 nguyen 16 véi v6 han s6 nguyén duong n, Cho ay, a2 dy, F& cde hing s6 thuc, « Fa mot bign thuc, va 1 1 cos(ay + 2) + 5 cos(ay + n+ ye (ag + x) +eee + F(x) cos(ay, +) pet an =a voi La motsé Gia sit rang f(x) = f(22) = 0, chimg minh rang x2 nguyen nao 6, . Vai mai gid tri k = 1, 2,3, 4, 5hay xdc dinh cdc diéu kign cn va di cla a > O sao cho t6n tai mot tt dign véi k canh d6 dai bing a sao cho va 6 — k canh cbn Iai c6 0 dhi bing 1 Cho -/18 nita duimg tron dudng kinh AB. Cla motdiém duge lay tren + khiic v6i cdc diém A va B va D 1A chan cia dubng vudng g6c ha tit C xudng AB. Ching ta xét ba duimg tron 1a 7), 7, 7, cing tiép xtic véi duting thing AB. Trong so chiing, dung tron >; noi tiép trong tam gic ABC, trong khi dé duimg tony vais cling tigp xiie voi CD va véi 7, vamdi cdi nim & méi nita mat phing 6 bo 1a dung thang CD. Ching minh ring 71,72 va 7s ¢6 mot tigp tuyén chung, Cho n > 4 diém trong mat phang sao cho khéng cé ba diém nao thang hang. Ching minh ring ¢6 ft nhat ("5°) t gidc 161 ma ede dinh 1a bon trong s6 cae digm da cho. Ching minh ring véi moi sO thye 2, 2, 91, 2. 21, 119) — 22 > 0, L242 — 23 > 0, tacd bat dang thite 8 (1+ 2) (i+ 2) = (1 +22) 2, Viz, > 0,29 > 0, Hay xc dinh digu kign cfin va dii dé dng thie xy ra, IMO: Céchai thi Olympic Todn quéc té 25 2.12 Lan thi mudi hai, tai Keszthely, Hungary,nam 1970 1 v Cho digm Mf nim trén canh AB clia tam gide ABC. Ki higu ry, ra var Ia bain kinh cla cc duémg trdn noi tigp cdc tam gite AAMC, ABMC va AABC. Ki higu qi,q2 va q Ja ban kinh cia cdc dudng tron bang tigp cita cée tam gide dé twong ting v6i g6e ACB. Ching minh ring ae Cho a, b van Ia cde s6 nguyén Ién hon 1. Xét céc 86 Ani, Ans Bui Va By IA cde 86 thod man c: Tn # 0on1F0 trong dé céc biéu dign cita céc $6 A,,-, By trong eo s6.a.Va An, By fa bigu dig trong co sib. Chimg minh rang An —1 Bur, , Ao < FF if and only if a > o - Cie 6 thu ap, a),-++,dy,++-thod man diéu kign: 1 =a S$ a1 Sa + Say See Ce 865 0), ba... by +4 duge xde dinh boi 2 ( ) 1 1 = a ax ) VaR (a) Ching minh ring 0 evéi n dit Ién. Hay xc dinh tit ca céc sO nguyen dong n thoa man tinh chat: tip hop {n,n + 1.n+2,n-+3,n-+4,n+5} 6 thé chia thinh hai tap hgp thoa man tich cila ede 86 trong hai tap hopd6 bang nhau. Trong ti di¢n ABCD 6 géc ZBDC = 90°. Gia siting chin dutmg cao duge ha tir D xuéng mat phang (ABC) tring véi tryc tam ciia tam gide phing AABC. Ching minh rang (AB+BC+CA)’ <6(AD? + BD? +CD") Dau bing xiy ra khi ndo ? ‘Trong mat phing c6 100 digm sao cho khong c6 ba digm nao thang hing . Xét tt ci ee tam gide dugc 6 cae dinh 18 ba trongs6 cfc diém da cho. Chimg minh ring khong c6 qua 70% tam gic 1a nhon. IMO: Céchai thi Olympic Todn quéc té 26 2.13 Lan thir mudi ba, tai Zilina, Czechoslovakia, nam 1971 1. Ching minh ring khing dinh sau diing cho n = 3 van = 5 vans sai véi moi sé nguyén duvong n > 2 khic v6i hai trating hop dé: " N&u ay, a2, Gy 1a cdcs6 thu, thi (a, —a)(a, — a3) (c= a1) (aq—05)***(ay— ty) F+**F (0A) (Gq M2) 0 v P, v6i chin dinh Aj Ap, Xét mot da gide 16 da g minh rng c6 it nhat hai da gide trong s6 cae da gid trong chung. ic P; bang céch tinh tién cdc dinh A) t6i A; twong tmgi = 2, 3, Py +9. Ching cP, Ps 3. Chimg minh rang tap hop ede s6 nguyén c6 dang 2" — 3 véi ( ‘mot tp con v6 han mi hai so’ bst ki trong chéing a nguyén t6 cing nhau. 4. Tat cA céic mat cia ti dign ABCD 1a cée tam giée nhon. Ching ta xét tit c& cfc da gidc ding 06 dang XY ZTX duage xéc dinh nh sau: X Ta mot diém nam trén canh AB phan bigt v6i A va B: tuong ty Y, Z,T Ia céc diém trong cia cc canhBC, CD, DA tuong ting. Ching minh ring: (a) Néu ZDAB+ ZBCD # ZCDA+ ZABC thi cde da gide dudng c6 mot dutng di vi d6 dai bé nhat. (b) Neu ZDAB + ZBCD = ZCDA + ZABC thicé vo han céc da gic véi do dai ngan nhat c6 do dai chung 14 2AC sin(a/2),6 46 o BAC +ZCAD+ ZDAB. 5. Ching minh ring véi moi sé nguyén duong m t6n tai mot tap hop hu han S gdm cdc diém trong mot mat phing véi tinh chat: VGi mgi digm A trong S'c6 ding m digm trong S ma khong cach t61 A bang don 6. Cho A = (aij) voi Gj s6 nguyen khong am. Gia sit ring bat ctf phan tir ndo aj, tit nam trén dong 4 va edt j déu> nm. Ching minh rang t6ng cia tat cd cdc phan tit ca ma tran néi tren khong nha hon n?/2. 1,2, ...422) JA mot ma tran wudng véi 2.14 Lan thi mudi bon, tai Torun, Poland, nam 1972 1. Chitng minh ring tir mot tap hop gm c6 mudis6 ¢6 hai chit s6 trong h¢ thap phan, 4 thi moi ti gide noi tiep duye trong mot duémg tron. du c6 thé phan hoach thinh n tit gide ma mdi tt gide déu c6 thé ndi tigp duge trong mot dudng tron. IMO: Céchai thi Olympic Todn quéc té 2 3. Chomvanla c s6 nguyen khong am. Ching minh ring (2m)!(2n)! mink nl 1 mot s6 nguyen, (Trong dé ta quy wée 0! = 1), 4. Hay tim tat cd ede nghiém (01,9, ,."44 275) cla he bait phutong trinh sau day 0 0 0 0 0 INIA IA IA lA trong d6 2, 23,03, :%4,.25 Ia ede 86 thye duong. 5. Cho f vag IA céc him gid tri thye xéc dinh tren tap tat c& ede s6 thue « va thoa ‘man phuong trinh him Flw+y) + f(xy) v6i moie,y € R. Chimg minh ring néu f(c) khong déng nhat khong, v [F(o)| <1 v6i moix € R thita 6 |g(y)] <1 véimoi y € R. 2F(x)g(y) 6. Cho bén mat phing phan biét song song, ching minh rang tén tai mot tit dién déu ma méi dinh ciia né thude dting mot trong bén mat phing néi trén. 2.15 Lan thi mudi nam, tai Moscow, U.S.S.R, nam 1973 1. Cho mot duimg thing £ trong mat phing va mot diém O nim tren n6, Nguoi ta dung cae vector don vj OF, OP, ...,OP, sao cho cae diém ngon cia cic vector nay Ia Pi, P,.... P, nam trong cing mot nita mat phang c6 bd 12 dudng thing £ Ching minh ring néun 1a so nguyen duong lé thi OP, + OP. +++ +0P, 21 | durge ki higu la modul (hay do dai) cha vector O17. 2. Hay xéc dinh xem lieu c6 tén tai hay khong mot tap hgp hitu han M cde diém trong khong gian ma khéng nam trén cing mat phing thoa man véi hai diém A va B bit Ki ciia M chiing ta déu c6 thé chon ra dugc hai digm ¢ va D cia M sao cho cdc dutmg thing AB vaC'D song song ma khong tring nhau. IMO: Céchai thi Olympic Todn quéc té 28 6 thue tho’ man: phuong trinh 2! -ar' + bx? +ar4+1=0 nhait mot nghiem thuc. Véi tat ec: tri bénhat cla a? + I. 86 thye (a,b) nhu vay hay tim gid 4, Mot ngudi linh muGn kiém tra suv nhanh tri ciia minh bing céch thye hign mot nhigm vy trong mot ving c6 hinh 1a mot tam gis Ban kinh hoat dong cla chige may dd cia anh ta 1a bing mot nita chan dung cao cita tam gide. Neuwdi Iinh tri dang ding fa mot dinh cia tam ho va bit diiu cong vige. Héi ing con duémg ma ngudi linh nén chon dé cé6 thé di mot quang dubng ngin nhat ma vin hoan thinh nhigm vu ciia anh ta? 5. Cho G a mot ho cae ham khéc hang s6 véi bién thye x c6 dang F(c) =ar-+bé d6 a vab la cde s6 thue. sao cho G tho miin céc tinh chat sau day: (a) Néu f vag thude vio G thi tich hgp thanh 9 0 f cfing thude vao G. (b) Néu f thude G thi ham nguige ciia n6 cing thude G. (©) Véi moi f thuge vao G thi tén tai mot s6 thue «tha man f(xy Ching minh rang t6n tai mot s6 thyc A tho’ man f(t) = & véi moi f thude va0 G. 6. Cho a, 195.24 ty Han $6 duemg va dat q 14 mot sO thyc cho true tho man 0 < <1. Hay xac dinh n $6 thue Oy, boy... Dy thod man (a) ag < dy véi moi k= 1,2. beet Le (b) q< = < = v6i moi eG 1+ (©) be # by beet by < Fe 2.16 Lan thi mudi sdu, tai Erfurt, West Germany, nam 1974 [TT] Ho va ten Lop | Trutng ‘Huy chuong T_[Hoing Le Minh 10 _[DHKHTN Ha Nor] 38 HCV 2 [Va Dinh Hoa 10_[ DHSP Ha Noi 31 HCB 3 [Ding Hoing Trung | 10 [DHKATN Ha Noi | 28 HCB 4_|Ta Hong Quang T0_| DHSP Ha No 27 HCD 35 _[Neuyén Quée Thing [10 [DHKHTN Ha Noi | 22 IMO: Céchai thi Olympic Todn quéc té 29 1. Ba nguoi / B va cing choi mot trd choi nhu sau: C6 ba edi lf bai, tren moi Id ba £6 ghi m6t s6 nguyen, Ba s6 nguyén dé ta kihigu lap, q,r tho man 0

2 > 2 Yn Ching minh ring néu 2, 20,+*+ 2, 18 mot odin vicita yi. yos** «Ya th ws Soe - Si = v Cho a, 2,4, véimoi p Ja mot day tang v6 han céc s6 nguyén duong. Ching minh rang 1 déu t6n tai v6 han cdc s6 a,,, thod man yy = Ly + Yl v6i7, y lacde sé nguyén duong va q > p. 3. Tren céc canh cia mot tam gide tuy ¥ AABC cfc tam gide AABR, ABCP, ACAG duge dung vé phia ngoai sao cho ZCBP = ZCAQ = 45°,ZBCP ZACQ = 30°, ZABR = ZBAR = 15°. Ching minh rang ZQRP = 90° va QR=RP. 4, Ta ki higu A 1a téng cdc chits6 ctta s6 4444 viet trong he thap phan, B 1d tng cdc chit sé cla sé A. Hay xc dinh téng cdc chitsé cla sé B. oan hoe sinh Viet Nam nam 1975 doat hai huy chyong Bac va ba huy chyong Déng. Tuy nhién theo nhimg thong tin ma Thay gido, PGS. TS: Neuyén Hoi Nel thinh vien ciia doi tuyén Viet Nam tham dur ki thi IMO1975, cung cap, thi Viet Nam nam d6 1a mot huy chuong Bac va ba huy chong Déng. Vi thé chiing toi tam thé cchuta dua ra thong tin vé thanh vién di tuyén va thanb tich cba doi nim 1975 va se ‘i ligu nay trong thai gian sm nhdit 6 thé, Nhan day, ching 181 cam on chi t6i PGS.TS. Nguyén Hoi Nghia vé nhiing thong tin quy bau da cung cap cho Dién dan. Phan Vii Diém Hing da tra thinh hoc sinh nit dau tien ciia Viet Nam tham du’ ki thi IMO. IMO: Céchai thi Olympic Todn quéc té 31 5. Hay xéc dinh xem ligu c6 thé tim duge 1975 don vi hay khong ma thoa man khoaing céch gi im trén dung trdn véi ban kinh hai diém bat ki déu 1a 86 hita ty. 6. Hay xc dinh tat ef cdc da thite P hai bién thod man cdc tinh chat sau day: (a) V6i mdi so nguyen duong n va moi s6 thyc fr, y ta 66 P(te, ty) =t"P(a,y) (b) véi moi so thue a,b, ethi P(b-+0,0)+ Plc+a,b) +Pla+b,0) © PUL,0)=1 2.18 Lan thit mudi tam, tai Linz, Austria, nam 1976 TT [Mo vaten Lop | Tring Huy chuong T_[Nauyén Thi Thigu Hoa 10 [DHKHINHaNGi | HCB 2_[Te Ngoo Chayen T0_| DHSP Vinh HCD 3__[ Nauyén Hing Son 10 HCD a_[Lé Ngoc Minh 10_| DHSP Ha Ni HCD 5_[ Ha Huy Bing 10_| DHKATN Ha Noi 6_[ Nauyén Van Hanh 7_[ Phan Thanh Dién DHSP Ha Noi & [Le Hai Khoi DHKHTN Ha Noi loi véi dign tich bang 32 tng do dai ciia hai canh doi ving 16, Hay xac dinh tat cd ede gid tri c6 thé do dai clia 1. Trong mat phang cho tit dign va mot dudng chéo duting chéo cdn lai. da thite P,,(2c) thoa man P, (x —2va Pix) = P,(Pja(x)) v6i Ching minh ring véi moi so nguyen duong i thi nghigm cia phuong tinh 18 cac s6 thye va doi mot khdc nhau. 3. Mot chige hop hinh chit nhat duge chia thank céc hinh lap phuong don vi. Gid sit rang ngudi ta c6 thé dat cdc hinh lap phuong c6 thé tich bang 2 vao trong hop sao cho ec canh cita cdc lap phuong dé Ia song song véi cdc canh cita hop va ring: 4. Hay xc dinh gid tri lin nhat e6 thé cit tich edes6 nguyen duong ma e6 tng bing 1976. é 5. Xét he pphuong trinh voi g = 2p dn 2y,.r9,.--4-r¢ Sau day: ayn, + ay2t2 + Ayr) + aypty +++ Opty + Ayaty +--+ + dyytq 6 46 cc hg s6 ay thude vao tap {—1, 0,1}. Ching minh ring hg néi trén c6 mot nghigm (c),.22,*** 27) thoa man (a) Tata cde s6 x; déu nguyén (j = 1 +4). (b) Téntai it nhat mots6 j max; 40. (©) |x;| 0 véi moi sé thue 0 thi +P <2va A+B < 1, 5. Cho cdc sé nguyén duong a, b thod min a? + 6? chia cho a+ c6 thuong Ia q va 86 dura r, Hay xc dinh tat ed ede cap (a, b) thod man q? + r= 6. Cho /(n) & mot him xée dinh tren tp cf s6 nguyen duong va e6 gid ti trong tap 46, Ching minh rng néu F(n +1) > F(F(n)) ‘véimoi s nguyen dung n thi f(n) = n v6i moi s6 nguyen duong n. 2.20 Lan thit hai muoi, tai Bucharest, Romania, nam 1978 TT [Mo va ten Trung, Huy chuong T_[VoKim Tuan DHSP Ha Noi 30 HCB 2_[Neuyén Thanh Tung | DHSP Ha Noi 29) HCB 3_ [D6 Dire Thai DHSP Ha Noi 25, HCD [Ho Dinh Duin ‘Quée hoc Hue 24 HCD 5_[LéNhw Duong. THPT Thai Phign, Hai Phong | 24 HCD © _[Neuyén Twin Hing _| DHSP Vinh 23 HCD 7_[Neuyén Hong Thai _| THPT Chu Vin An, Ha Noi_| 23 HCD &_[Neuyén Trung Ha | THPT Chu Van An, Ha Noi_| 22 HCD 1. Cho cde so nguyén dong m va n voi 1 < m 2 sao cho tén tai mot tap hgp gdm cé n sé nguyén duong lién tiép thoa man s6 1én nhat trong tap dé 1a mot ude cla boi chung bé nhitcila tat c4 n — 1 s6 cdn la 4 tr] Id nat cla m + n3, 6 ., 1981} va (n? —mn—m dé m va n Ia cde so nguyen thod (b) Véi nhimg gi tri n > 2 ndo macé ding mot tap ¢6 tinh chat n6i tren, 5. Ba duing tron déng quy tai mét diém O va nam trong mot tam gide cho truéc. MGi dung trdn tigp xtic véi mot cap canh cia tam gi yg minh rang tam dung trdn noi tigp va ngoai tigp cia tam gic néi trén va diém O la thang hang. 6. Cho mot him s6 gid tri thue f(x, y) hai bién thod man (@) fOy)=y+ls (b) f(w@+1,0)=f(w,1)s ©) Fetiy +) = fe fletly))s véimoi sO nguyen khong am 0, va f(9999) = Hay xac dinh gid tri ciia f(1982) 3. 2. MOt tam gide khong can AA Ay Ay véi dé dai cdc canh 1a ay, a», as (trong dé a; 1 doi dign véi dinhA;, v6i moi i = 1, 2,3). Voi mdi # = 1, 2,3ta gol M, 1 trung diém cita doan thing ¢6 do dhi a; va T; la diém tigp xtc cita dung tron ndi tig tam gide véi canh a;. Déng thoi ta ki higu S, Ia diém doi xting ciia T; qua phan gidc trong ciia g6c A;. Chiig minh ring cdc dung thing M,S;, MzSo Va MS 1h déng quy. 3. Xét cde day hitu han {17,} cde s6 thye durong thod méin cdc tinh chat sau: a = Land forall i> 0,24) <2 (a) Chimg minh rang véi moi day nhu trén déu cd mot sé nguyén n > 1 tha man 4. (a) Chimg minh ring néu n I mots6 nguyen duong thod man phuong tinh, r= Bry +y8 n c6 mot nghigm nguyén (7, y), thiné cé ft nhait ba nghigm nguyén, (b) Chimg minh ring phuong trinh trén khong c6 nghigm nguyén khi n = 2891, 5. Céc dung chéo AC vaACE cia mot luc gide déu ABC DEF duc chia trong bai cde diém chia M va V tuong ting sao cho 4M _CN AC ~ CB hing, Hay xéc dinh r néu B, M va Nt 6. Cho $ 1a mot hinh vudng véi canh d6 dai bing 100 va Z 14 mot dudmg khong ty cét ben trong hinh vudng d6. I. duge thanh lap bai cc doan thing Adi, Arlo, «+4 An-1Aq v6i Ag # Ap. Gil sit rang v6i moi diém P trén bign cla S cé6 mot diém i +h P mot doan khong lén hon 1/2, Ching minh rang c6 hai digém X va ¥ trong I. sao cho khoiing céch gidta ching khéng lén hon 1 va do dai cua phiin duimg Z ma nim gia hai diém X va ¥ khong bé hon 198. é 37 IMO: Céchai thi Olympic Todn quéc té 38 2.24 Lan thi hai muoi tu, tai Paris, France, nam 1983 TT Lop | Tring Huy chuong T g 12_| DHSP Ha Noi HCB 2 Nauyén Van Luong [12 | Qude hoc Hue HCB 3_[ Trin Nam Ding T1_[THPT Phan Chau Trinh, Di Ning | __HCB [Pham Thanh Phuong [| 11_|DHSP Ha Noi HCD 3_[Ho’ng Ngoc Chign | 12 | Que hoc Hu HCD © [Neuyén Viet Ba T_[THPT Thai Phign, Hai Phong, HCD v }. Cho tam gidée AABC. Hay xée dinh tat duong va thod man cfc him f xc dinh ten tap c&e s6 duong md nhan gié tri thye diéu kien sau: (a) (f(y) = uf (a) voi moi so duong x,y; (b) f(x) 3 0asz— 00. Cho A 1& mot trong hai giao diém c tam O, va O>, twong ting. Mot trong hai dudng tron déng phing C, va C, véicde : tigp tuyén chung t6i cde duimg tron tig xiic v6iC; tai P, va C, tai diém P, Iai tigp xtc voi C; tai diém Q) va V6i C, tai diém Q. Cho M, cia doan thang P,Q) va Mz la trung diém ciia doan thing P,Q. Chimg minh ring 20; AO) = 2M, AMp. Cho cde s6 nguyen duong a, b vac saocho Ching minh rang 2abe— ab — be dign duge dudi dang xbe +yea + 2ab, 86 bit ki déu nguyen t6 cling nhau. n Ién nhat ma khong thé bigu ic 86 nguyén ‘hone am BC vaCA (tinh cdc cdc diém mit) fap € thnh hai tap con 19% nhau thi c6 ft nhat mot trong hai dinh cia mot tam gide vudng. Hay ching minh céu tra loi Cé thé chon duge hay khong 1983 86 nguyen duong phan bidt, tat cd bé hon hoe bang 10°, ma khong ¢6 ba 6 nao trong chiing lai ap thanh mot ep s6 cong? Ching, Cho a, } vi cla do dai ba canh ciia mot tam gide. Ching minh ring a’b(a—b) + Bc(b— 0) + ale—a) = 0. Hay xc dinh xem khi nao thi dang thite xay ra. IMO: Céchai thi Olympic Todn quéc té Lan thit hai muoi nam, tai Prague, Czechoslovakia, nam 1984 STT | Ho vaten Trung, Diém | Huy chuong T_| Dam Thanh Son DHKHTN Ha Noi a HCV 26 Quang Dai DHSP Ha Noi HCB 3_[ Neuyén Van Hung HCB 4_[ Ngo Thac Anh HCD 3_| Vo Thanh Ting HCD ©_| Nguyén Thi Minh Ha | THPT Chu Van An, Ha Noi HCD 1. Céc 86 thyc khong am ar, y, 2 thod min x + y+ 2 Ching minh rang OS aytyetzr—2eye < v Hay xéc dinh tat cA kién sau: ‘Ap sO nguyén duong (a, )) thoa man déng thoi hai diéu (a) ab(a-+b) khong chia hét cho 7. (b) (a +6)" <0? ~ 7 chia hét cho 3. Trong mat phing cho hai diém phan bigt O va A. Véi mdi diém X cia mat phiing khée véi O ta ki higu a(X) 18 d9 do cia gc gitta OA va OX theo don vj radian, theo huténg nguige kim dng hé (OA(0 < a(X) < 27). Kf higu C(X) 1a dudng trdn tim O véi bén kinh IAOX +0(X)/OX trong mat phang duge t6 ‘mot miu trong mot s6 miu cho truée. Ching minh ring t6n tai mot diém Y ma a{Y) > O sao cho céc miu cla né xuat hign tren dudmg tron C(Y’) 4. Cho ABCD la mot tit gidc l6i thod man dudng thing CD tiép xtic vi dudng tron. duing kinh AB. Chimg minh ring dung thang AB la mot tigp tuyén t6i duéng tron dudng kinh C’D khi va chi khi cdc dutng thing BC’ va AD Ia song song. 5, Taki higu d Ia t6ng do dai cia cae duimg chéo ciia mot da gide phing e6n dinh (n> 3), va p Ia chu vicita da gide 46. Ching minh ring w3< He [e[PAY 2, 6 46 [| dug 6. Cho a,b,c va d la ede s6 nguyen 1é thod min 0 1 Tay = Ching minh ring t6n tai ding mot gi tri ciia v1 d8 saocho O 4. Ngudi ta dung mot tap hap c&e diém P;, P,P sao cho Pi. Ia dnh eta P, qua phép quay vi tam Ta Avy wéigée quay 120° theo chiéu nguge kim déng h6 (v6i : thitam gide AA) Ay ds 1a tam gi . Mdi dinh cia mot ngii gidc déu duoc gin mot s6 nguyén sao cho téng ciia cd nam 56 d6 la s6 durong. Néu ba dinh lien ti¢p duge dan cdc s6 la x. y. 2 wong img va y< thi tage hhign mot toan tirsau da 2+y tuong img. Mot t a Ei coit ait mot trong nam s6 1a am. Hay xdedinh xem ligu qui trinh tren c6 thé dign ra va két thic sau mOts6 hitu han bude hay khong, buy chutong Vang tai mot Ki thi eiia IMO, © HaDuy Hing 28" IMO, tai Havana, Cuba, nam 1987 42 Hib) Op eee 27'h International Mathematical Olympiad Warsaw, Poland Negdy thi thie hai Ngdy 10 thing 07 nam 1986 Sem OY OR aE Ee 4. Cho 4 va B la hai dinh lien tigp cud mot da gide phing déun canh (& a6 n Goi O Ih tam ca da gide d6, mot tam gide ANY Z chuyén dong trong mat phing ban du AOAB, sao cho cdc dinh Y va Z tru tren . Hay xc dinh qu tich bien cia da gi cha diém X. 5, Hay xc dinh tit c& cfc ham f xéedinh trén tap cfc s6 thye khong am va nhan cée gi tri thuc khong am, sao cho: (a) f(xf(y))\ f(y) = f(a +y) véi moi x, y > 03 (b) FC (©) fe) 40vei0Sr<2. 86 cfc diém tring va dé lun song véi mot trong hai truc tog do thi sur chénh Ie Jon hon 1 2.28 Lan thit hai muoi tam, tai Havana, Cuba, nam 1987 STT | Mo va ten Lop | Tring Diém | Huy chuong T [Tran Trong Hang | 11 | DHSP Hi Noi 38, HCB 2__[Nauyén Van Quang | 12 | THPT Lam Son, Thanh Hod | 30 HCD 3_[PhanPhuong Dat [11 | THPT Ha NorAmsterdam | 29 HCD 4_|Pham Trigu Duong [ 12 | THPT Hi Noi-Amsterdam | 28 HCD 3_[Doin Quée Chign _[_I1_[ DHKATN Ha Noi 25 HCD. 6 _[Neuyén Haw Tuan | 12 [DHKHTN Ha Noi 22 HCD © HaDuy Hing 28" IMO, tai Havana, Cuba, nam 1987 43 Hee ER OOOO HHH 28'h International Mathematical Olympiad Havana, Cuba Negay thi thie nhai Ngdy 10 thing 07 nam 1987 Siberian OOO OO HebHESHRESH 1. Chom la mot s6 nguyen duong, Véi mdi s6 nguyen khongam k ta ki higu pp) 1a s6 céchosin vj cia tap hgp {1,2,-.-, 7}, ma e6 diing k diém c6 din, Chimg minh ring Yok pall) & Chai ¥. Mot hodn vif cila tap hgp S 1a mot song dnh di tir vio S. Mot phain tir i € S duoc goi la diém c6 dinh cia hodn vi f néunhu f (i) =7. v Cho tam giée nhon ABC, dudng phan gide trong ciia tam gidc tir A ct doan thang BC tai diém 1 va cét dug tron ngoai tip cia tam gide ABC & dim thit hai NV. Tir diém L ta ké cdc dutng thing wuong gée véi AB va AC, eft ede dutng thang nay twong ting 6 cdc diém K va M. Ching minh ring tt gide AKNM va tam gide A ABC 6 cing dign tich, 3. Ch +p lacdc 86 thyc thod man rib ajytee ta Ching minh rang v6i moi s6 nguyen k > 2 thi 6 cdc sOnguyén a4, a2, +++5 On, khong déng thai bang khong sao cho |a;| < k — 1 véi moi d va (k= Dva layay+arey bes bata] < Heer OOO VHHEEIHEEHIEE 28th International Mathematical Olympiad Havana, Cuba Ngay thi thit hai Ngay 11 thang 07 nm 1987 einer rEHHE OOO OO HebHESRESH 4. Ching minh ring khong t6n tai mot ham /' nao tir tap hgp cdc sO nguyen khong, am vao chinh né thoa man f(f(n)) =n + 1987 véimoi n. © Ha Duy Hing 29" IMO, tai Canberra, Australia, nam 1988 44 5. Cho mld mot s6 nguyen khOng bé hon 3, Ching minh ring 6 mot tap hgp n dim trong mat phang sao cho khoang cach giifa hai diém bat ki la mot sé vo ty va méi tap ba diém bat ki déu xdc dinh mot tam gidc khong suy bién ma dién tich cia né Ja mot sé hitu ty. 6. Cho 7 1a mot s6 nguyen Ién hon hay bang 2. Ching minh rang néu k? + k+ nla. MOL SO nguyén t6 véi moi sO nguyen & thod min 0 < k =< y/n/3, thik? +k+n 146 nguyén 16 véi moi 86 nguyén f thod min 0< k rv. Lay P 1a mot diém cé dinh trén dung tron bé va B 1a mot diém bién thién tren duing trdn Ién, Dug thing BP cat dudng tron lon & diém thit hai C. Duong thang £ vudng gée v6i BP tai diém P cit dung tron nhé tai diém thit hai A (neu hut dung thang ¢ tip xtie véi dutng tron nhé thita xem A= P). Atri c6 thé cita bigu thite BC? +CA’ AB? (a) Hay xc dinh tap tat ic dinh qu tich trung diém cilia doan thang BC. (b) Hay "Tran Trong Hang la hocsinh Viet Nam dau tien hai Lin tham dit IMO. © Ha Duy Hing 29" IMO, tai Canberra, Australia, nam 1988 45 2. Cho n 1A mot s6 nguyén duong va Ai, B.Gia sit ring + Any) Rede tap con ciia tap hop (a) Méi A; c6 diing 2n phan (b) Méi A:.A; (1 27. 6. Cho hai s6 nguyén duong a vib thod min ab +1 18 uée ela a? + 6?. Chimg minh ring : e+e ab+1 1a mot s6 chinh phuong, © HaDuy Hing — 30" IMO, tai Braunschweig, West Germany, nam 1989 46 2.30 Lan thit ba muoi, tai Brunswick, West Germany, nam 1989 TT [Ho va ten Lop | Trutmg Diém | Huy chuong T_[Dinh Tiga Cuing_[ 12 | DHSP Ha Noi a2 HCV 2 [Ngo BioChau__| 12 _|DHKHTNHa Noi 40 HCV 3_| Boi Hai Fimg, T_[DHKHTN Ha Noi a HCB a juy Minh 12 | DHKHTN Ha Noi 27 HCD 5_| Trin Trong Thang | 12_| NK Tran Pho, Hai Phong 21 HCD. 6 [Doan HongNghia | 12_[THPT Le Hong Phong, HCM | _19 HCD wine Heiser QOD OD eH SEHS 30! International Mathematical Olympiad Braunschweig, West Germany Ngdy thi shit nhai SecA Tee 1. Chimg minh ring tap hop {1, 2, nhau cita cdc t4p con A; (i = ién duge duéi dang hyp ri (a) Mé (b) Téng ciia tat ca cc phan tir trong méi tap A; 1a bang nhau. ip Ay chifa dting 17 phn ti, 2. Trong tam giée hon AABC dubng phan gic trong xuat phat ti dinh A ct dudng tron ngoai tip tam gidc d6 és dim thit hai Ay. Cac diém By va C; duge xéc dinh tuong ty. Kf higu Ay 1a giao diém cia dudng thing AA, véi duémg phan giéc ngoai ciia cde g6e B vaC, Cac diém By va Cy duge xd dinh twong ty. Ching minh, ring: (a) Dign tich cfia tam gide A Ap ByCh gp doi dign tich cia le gige AC, BA\CB Ngo Bio Chau trd thanh hoe sinh Viet Nam th hai hai Ian tham dur IMO, va anh cing [a sinh dau tien doat hai huy chvong Vang IMO. Hien nay anh van dang gitt ki luc [a hoc sinh Viet Nam 6 tong s6 diém doat duc 6 céc ki thi IMO cao nhat 42 + 40 = 82 diém. "Dinh Tién Cutng 18 hoe sinh dau tién cita Khoi THPT Chuyén Dai hoc Su pham Ha Noi doat s6 diém t6i da tai mot ki thi IMO. "Tran Trong Thang 1a hoe sinh dau tién cita ngOi nha NK. Trin Pho tham du mot ki md ra mot trang sirhao hing ciia cho ngoi trubng rat non tré nay (thanh lap nim 1987). 8 Nam 1989 fi nam dang nh ddi tuyén Toan quéc gia Viet Nam, lan dau tién doat hai huy chong Vang trong mot ki thi ¢ IMO. hi IMO, © Ha Duy Hing 30 IMO, tai Braunschweig, West Germany, nam 1989 47 tich ciia tam giée AA BoC khong bé hon bén Kin dign tich ca tam gidc AABC. (b) 3. Chon va k Ia ce s6 nguyen duong va S 1a tip hop gém n diém trong mat phing thod man (a) Khéng c6 ba diém nio ciia $ Ia thing hang; (b) Voi bat ki digm P nao ciia $ thi déu 6 it nhat k dim cia S cach déu diém P. Ching minh ring: kehivin Heiser QOD OD eH SEHS 30! International Mathematical Olympiad Braunschweig, West Germany Ngdy thi thit hai Seba! es 4. Cho tt giée 161 ABCD thod man cdc canh AB, AD, BC thod min ding thie AB = AD + BC. Gia sit ring tn tai mot diém P ben trong tif gidc cach duimg thang C’D mot doan bang hi sao cho AP = h + AD va BP = h+ BC. Ching minh ring: t.34, 4. Vhn~ VAD © VBC 5. Ching minh ring véi mdi sé nguyen duong n déut6n tai n s6 nguyen duong lign tigp sao cho khong c6 86 nao trong ching Ia mot luy thira ca mot sé nguyen 16. 2n},6 db m 1A mots6 nguyen 6. Mot hodin vi (1r1,:r2,---, Pan) cla tap hgp {1, duong, duge goi 1a 66 tinh chat P néu [ary —.r5..1| vi it nhat mot chi sé i thudc fl, Qo. 201} ‘Ching minh rang, véi mdi sé nguyén duong n thi sé cdc hodn vi c6 tinh chat P nhiéu hon s6 céc hodn vi khOng 6 tinh chat dé. © Ha Duy Hung 31° IMO, tai Beijing, People’s Republic of China, nam 1990 48 2.31 Lan thit ba muoi mot, tai Beijing, People's Republic of China, nam 1990 TT [Ho va ten Lop | Trutmg Diém | Huy chuong T_| Pham Xuan Du 12_[DHKHTN Ha Noi 2 CB 2_[Wa Xuan Ha 12 [| THPT Lam Son, Thanh Hod [19 HCD 3_| Phan Thi Ha Duong | 12 [THPT Ha Nor-Amsterdam [19 HCD [Le Trdng Lan 12 [DHSP Ha N 16 HCD 35_[ Wai Hoan; T_[THPTChuyen Thai Binh__[_15, 6 [Fa TL [DHRATN Ha Ne i Sincere DOU OO HebHESHRESH 315t International Mathematical Olympiad eijing, People's Republic of China Ngay thi shit nhai Ngdy 12 thing 07 nam 1990 eine HeHeHE OOOO O Habre 1. Cac day cung AB vi CD cia mot dung tron edt nau ta diém E ben trong ding tron, Gi sit rang M/ [a mot digm trong cia doan thing EB. duing trdn di qua cdc diém D, E va M cit duong thang BC va G tuong ting. Gia sit rang AM AB hay tinh v Cho n > 3 1a mot sé nguyén va xé gémcé 2n = 1 diém nam trén mot duing tron. Gia sir rang c6 ding k diém duge t6 miu den. Mot cach t6 miu nhut thé duge goi 1a 147 néu nhurcé it nhat mot cap diém miu den sao cho phiin trong 3. Hay xi dinh tated ede 6 nguyen n > 1 thod man 41 nm 1a mot s6 nguyen. © Ha Duy Hung 31° IMO, tai Beijing, People’s Republic of China, nam 1990 49 Hee ER OOOO HHH 315t International Mathematical Olympiad B 2, People's Republic of China Negdy thi thie hai Ngdy 13 thing 07 nam 1990 eine reHHEE OOOO O HebHeSn EH 4. Ki higu Q* [a tip tat cd cdc s6 hitw ty dutong. Hay dug mot ham f : Q* + Q thod min r 5 f(ef(y)) === yu véimoi x,y in@ 5. Hai ngudi choi A va B cing choi mot trd choi nhu sau: ban déu ta cé mot sé. nguyen no > 1, hai ngt luot chon cdc 86 nguyen m1, no, Ms, ...theo thit ty cia cde quy tic choi dui d cho. 1a lu thira nguyén duong cita mot s6 nguyen 16. Ngudi choi A sé chién thing néu chon duge s6 1990; ngudi choi B sé chién thang néunhu chon duge s6 1. Hai rang véi nhiing 6 rp ndo thi (a) A cé mot chien thuat dé chién thing? (b) Bcé mot chién thuat dé chién thing? (c) Khong ai c6 chién thuat dé chin thing? 6. Ching minh ring t6n tai mot da giéc 1990 canh 16i tho’ man ec tinh chat sau day: (a) Tate cée géc 12 bing nhau. (b) DO dai cia 1990 canh chia da gidc 18 cdc 86 17, 2”, 3”, ..., 1990? theo mot thi ty no dé. © Ha Duy Hing 32" IMO, tai Sigtuna, Sweden, nam 1991 50 2.32 Lan thit ba muoi hai, tai Sigtuna, Sweden, nam 1991 TT [Hovaten Lop [Trig Diém | Huy chong T_[DoNgoc Minh | 12_[THPT Lam Son, Thanh Hod 38 HCB 2_[Nguyén Viet Anh | _12_[ DHSP Ha Noi 38 CB 3 i 12_[DHKHTN Ha Nor 36 HCB a Huy Ta 12_[ THPT Phan Boi Chau, Neh@An |_31 HCB 5_[ Nguyen Hai Hi _[ 12 _[DHKATN Ha Noi 27 HCD © [Ngo Dien Hy 12_[THPT Lam Son, Thanh Hod ZI HCD . Cho n > 6 la mot sé nguyén va ay.02,.--,05 Lata Hee OOO USHERS EEE 324 International Mathematical Olympiad Sigtuna, Sweden Ngay thi thie nha Ngay 17 thang 07 nam 1991 ‘Thdi gian lam bai: 42 gia Hee OOOO SHEE EEEEEE Cho tam gide A ABC véi I 14 tam cia dudng tron ngi tigp tam gide. Céc pha cia cde g A, B va C cat céc canh d6i dién 6 A’, B! va C' tuong img. Ching, minh ring 1. ALB Cl 4 Als BB'+CC" cde so nguyén duong bé ngnéu hon n ma nguyen t6 cting nhau v6i r. Ching minh Gz Gy =A3 — Az =ay— a4 > 0, thin [A mot s6 nguyen t6 hoje phai Ia mot lu thita cia hai. Cho $= {1,2,3,..., 280}. Hay xdc dinh sé nguyen duong bé nhitn sao cho méi {ap con ¢6 n phan tir cla S déu chifa nam so ma doi mot nguyén 16 ciing nhau © Ha Duy Hing 32" IMO, tai Sigtuna, Sweden, nam 1991 51 Sincere DOU OO HebHESHRESH 324 International Mathematical Olympiad igtuna, Sweden Negay thi tht hai Negay 18 théng 07 nam 1991 ‘Thdi gian lam bai: 42 gia eine rernEE OOO OO Habre ESHR 4, Gia sit ring G 1a mot dé thi lign thong véi & canh, Ching minh ring c6 thé din k sao cho tai méi dinh ma thudc vao mot hodc hai can, > canh dé Fa bling 1 nhan ede canh 1,2 ude chung lén nit ciia ede s6 nguyén duge dn va 5. Cho tam gife ABC va P 1A mot diém trong cia tam giée ABC. Chitng minh rang it nha mot trong cde gée ZPAB, ZPBC, ZPC A c6 s6 do khong vuot qué 30°. 6. Mot day hitw han 9,2), 2, ic SO thuc duge goi Ia bi chan néu nhu cé mot hang s6 thye C sao cho |ar| < C_ véi moi 1 > 0. Cho truvie mot s6 thuc a > 1, hay dung mot day bi chan v6 hanary,.r),-r2,--» thod man ley ay|li-g|° >1 v6i moi cp chi sé nguyen dutong é,j © Ha Duy Hing 334 IMO, tai Moscow, Russia, nam 1992 52 2.33 Lan thitba muoi ba, tai Moscow, U.S.S.R, nam 1992 TT [Ho va ten Lop | Tring Diém | Huy chuong T_[Nguyén Xuan Dio | 12_[DHKHTNHa Noi 33 HCV 2_[ Neuyén Quéc Khanh | 12_| DHKATNHA Noi 26 HCB 3_[Nguyén Thanh Cong | 12_[DHKHTNHa Noi al HCB 4 [Neuyén Xuan Long | 12_[NK Trin Phi, Hai Phong | 21 HCD 3_[Nguyén Huy Cuting_[_12_[DHSP Ha Noi 18 HCD 6 [Neuyén Thuy Linh | 12_[DHKATNHa Noi 16 HCD v Sincere DOU OO HebHESHRESH 334 International Mathematical Olympiad ia Moscow, Ru Ngay thi shit nh Ngay 15 thing 07 nam 1992 ‘Thoi gian lam bai: 42 gio eine rernEE OOO OO Habre ESHR Hay xac dinh cdc sO nguyen a,b,c thod min 1 4 (b) Hay (c) Chimg minh ring tén tai v6 han s6 nguyén n thoa man $(n) =n? — 14. ic dinh mot sO nguyen m thod man S(n 2.34 Lan thit ba muoi tu, tai Istanbul, Turkey, nam 1993 STT | Ho va ten Lop | Trung, Huy chuong T_|Neuyén Chu Gia Vuong | 11 | DHKHTN HG Nor HCV 2_[Bii Anh Van TL_[THPT Lam Son, Thanh Hod HCB 3_[ Truong Ba Ta TL_| THPT Phan Boi Chau, Nghe An HCB 4_[Pham Hong Kien T_[DHSP Ha Noi HCB 35_| To Huy Quynh T_| THPT Chuyen Thai Binh HCB ©_| Pham Chung Thay T_[DHSP Ha Noi HCD © Ha Duy Hing 34!" IMO, tai Isiambul, Turkey, nam 1993 54 u v ™ Pham Chung Thuy Heiser QOD OD eH SEHS 34! International Mathematical Olympiad Istambul, Turkey Ngay thi thie nat Ngay 18 thang 07 ndm 1993 ‘Thdi gian lam bai: 42 gia eine rernEE OOO OO Habre ESHR 1" 452! 4-3, 6d6n > 14 mot sé nguyen. Ching minh 1u dign duge duéi dang tich cia hai da thifc khdc hang SO Cho da thie f( rang j() khong thé véi cdc he s6 nguyen, Cho D li mot digm nam trong tam gige nhon AABC tho’ man ZADB = ZACB+ n/2va AC+BD = AD+BC. AB-CD AC-BD (b) Chuwngs minh rang cdc tigp tuyén tai C' 161 cdc dung tron ngoai tip tam giée AACD va ABCD Ia wuong géc véi nhau (a) Hay tinh ty so . Trén mot ban cé v6 han ngudi ta thuc hién mot trd choi nhu sau. Bat du véi n? ming banh duge dat len trén ban cd ben trong mot bing con kich thie n xn gdm céc hinh vudng don vi cita ban c& xép ké nhau, moi miéng binh duige dat vao trong mot hinh vung. Mdi mot Kin thuc hign mot nuéc di trong tro choi Ia thyc hign ‘mot butée nhay tir mot hinh vudng c6 chia biinh theo huténg ngang hoac doc qua hinh vuong nim ké v6i hinh vuong dé va dat migng bénh vao ngay hinh vuong ke tig. Migng binh nto bi nhay quasé bi bd di. Tro choi sé két thiic néu nur chi con ving mot migng ban dat tren bin cés. Hay xac dinb tat c& cfc s6 nguyen duong n ‘mi trd choi c6 thé ket thiic v6i chi mot migng bénh cdn lai 6 trén ban, la hoe sinh nif dau tién va duy nhat cho dén nay cia Khoi THPT Chuyén Dai hoe Str pham Ha Noi duge tham du mot ki thi IMO. © Ha Duy Hing 34!" IMO, tai Isiambul, Turkey, nam 1993 55 Hib) Op eee 34! International Mathematical Olympiad Istambul, Turkey igdy thi thit nhdit Ngdy 18 thing 07 nam 1993 ‘Thoi gian lam bai: 42 gio beac OHI 4, Tir ba digm trong mat phing ta xée dinh m(PQR) ia ba duimg cao cia tam giée APQR. (Néu nhu cdc diém la thing hing, ching ta dat m(PQR) = 0). Ching minh ring véi cde diém A, B,C, X trong mat phing tuy ¥ ta 6: m(ABC) < m(ABX) + m(AXC) + m(XBC). 5, Tén tai hay khong mot ham s6 f: N+ N thoa man f(1) = 2, f(F(n)) =f(n)+n vdimoin EN, va f(n) < f(v +1) véimoin €N? 1n6 dang duge bat. G Ching minh ring: it ring thoi diém ban dau tat dén déu dang duge bat. (a) C6 mot s6 nguyen duong M(n) thod min sau M(n) bude thi tit ea cée d8n du duge bat len (b) Neu m 1a mot luj thira nguyén dutong cia 2 thi tac6 thé lay M(n) =n? =1 (©) Néun = 2 +1 v6i k nguyen duong nao dé thi chting tacé thé lay M(n) = n+l Cha y ring o day Ki higu s, thue chalt 1A bude thet, trong d6 chit cdi s & day 1A viet tit cia chir step te 1a buve, © HaDuy Hing 35!" IMO, tai Hong Kong, China, nam 1994 56 2.35 Lan thit ba muoi nam, tai Hong Kong, China, nam 1994 TT [Ho va ten Lop | Trutmg Diem | Huy chuong T_[Dho Hai Long, TL_[DHKATN Aa No |_4T HCV 2_[ Trin Ngoc Nam 12_|DHKHTN Ha Noi | 34 HCB 3__[ Neuyén Duy Lan 12_| DHSP Ha Noi Ey HCB 4_| Neuyén Quy Tu: 12 [DHKATN Ha Noi [33 HCB 5_[ Neuyén Chu Gia Vugng | 12 | DHKHTN HA Noi] 31 HCB © [To Dong Vi TL_[DHKHTN Ha Noi [30 HCB v Heiser QOD OD eH SEHS 35! International Mathematical Olympiad Hong Kong, China Ngay thi thie nha Ney 13 théng 07 nam 1994 ‘Thoi gian lam bai: 44 gia einer HE EEE OOOO HebHeanESHR Cho cdc s6 nguyén duong m van. Cho a1, 42,+4444m Ra cde phin tit khée nhau ciia tap hgp {1,2,...,} thod man néu a;-+ a; S) thoa man dong thai hai diéu kign sau day: (a) Fle + Fly) + xfly)) = yt F(x) +yF (©) vdimoi x va y thude S; !a mot him s6 tang ngat trén mdi khodng (—1;0) va (0;+-00) 6. Ching minh ring tén tai mot tap hgp A cdc s6 nguyén duvong thoa man cdc tinh chat sau day: Véi bat ki mot tp hgp v6 han S'nko cc s6 nguyen t6 déu c6 hai so dutongm € Avan ¢ A sao cho moi s6 déu Ia tich cba k phiin tir phan bigt 'S (6 d6 kk mot s6 nguyen Ién hon 1). 2.36 Lan thit ba muoi sdu, tai North York, Canada, nam 1995 STT | Mo va ten Diém | Huy chuong T_[NeoDic Tuan a2 HCV 2__|Dao Hai Long 40 HCV 3_[ Nauyén The Phuong 35 HCB 4__[Neuyén The Trung 35 HCB 3_[ Pham Quang Tuan 35 HCB © [Cao Van Hanh 12_[THPT Lam Son, Thanh Hod [33 HCB © Ha Duy Hing 36" IMO, tai Toronto, Canada, nam1995 58 117 Heiser QOD OD eH SEHS 36" International Mathematical Olympiad Toronto, Canada Ngay thi thie nha Neay 19 théng 07 nam 1995 ‘Thoi gian lam bai: 43 gia einer HE EEE OOOO HebHeanESHR 1. Cho bon digm A, B,C, D phan bigt nim trén mot dudng thing theo thit ty 46. Cae dug tron dudng kinh AC va BD giao nhau tai X vaY. Dudng thing XY cit dugng thang BC tai diém Z. Kf higu P 1a mot diém nam tren dudng thing XY khéc v6i Z. Dudng thing CP cat dudng tron dudng kinh AC tai diém C va M, va dung thing BP cit dung tron dutmg kinh BD tai diém D va N, Ching ‘minh ring cdc dung thang AM, DN va XY 1a déng quy. 2. Cho cde s6 thue dong a,b, thoi man abe = 1. Ching minh ring 1 1 @(b+ Ot Re +a) 3. Hay xe dinh tit ca ede s6 nguyen n > 3 sao cho t6n tai cde diém A\,..., Ay trong ‘mat phang sao cho khéng c6 ba diém nao thang hang va cdc s6 thuc r ra thoa man v6il CF 6. Cho pla mot s6 nguyen t6 16. Hai rng ¢6 bao nhigu tap hgp A c6 p phan tir va a tap con eta tap hgp {1,2,-.. 2p} ma tng ela cdc phan tir ciia A chia hét cho p? 2.37 Lan thit ba muoi bay, tai Mumbai, India, nam 1996 TT [Mo vaten Lop | Tring Diém | Huy chuong 1_| Ngé Dic Tuan 12_| DHKHTN Ha Noi 37 HCV 2_ | Ngé Bite Duy, 12_[ NK Tran Phi, Hai Phong 33 HCV 3 | Neuyén Thai Ha | 12 | DHKHTN Ha 31 HCV 4 | Pham Lé Hing T1_| DHKHTN Ha Noi 27 HCB 5_ [BS Quée Anh [11 DAKATN Ha Noi 18_| HCD 6_| Trinh Thé Huynh | 12 | THPT Lé Hong Phong, Nam Dinh | 9 18 192021 Hee OOO USHERS EEE 37!H International Mathematical Olympiad Mumbai, India Negay thi thit nhdt: 9 a.m. = 1:30 p.m. Ngay 10 thang 07 nam 1996 ‘Thdi gian lam bai: 42 gi HaeeeceseneE QOOOD eH Hem sEEES T’Ngo Dic Tuan tra thanh hoc sinh thit ba ciia Viet Nam hai lin doat huy chuong Vang tai IMO, t6 diém thém mot dau son chéi loi trong thar ang doi ca cdc Iép Ao cla trudng Dai hoc Tong hop Ha Noi nay la Dai hoc Khoa hoc Ty nhién, Dai hoc Quéc Gia Ha Noi. Ngo Die Duy la hoe sinh dau tien cua NK Tran Phii, Hai Phong dost huy chuong Vang tai ‘mot ki thi IMO. 2 Nam 1996 1d mot nam déng nhé cita doi tuyén Olympic Todn que gia cita Viet Nam, iy en dan hay cham Vang ong mot IMO. 2 Ldn thit hai trading DHKHTN Ha Noi co hai hocsinh doat huy chvtong Vang tai mot ki thi IMO. in © Ha Duy Hing 37" IMO, tai Mumbai, India, nam 1996 60 1. Cho true r 14 mot sO nguyen duong va mot bing hinh chit nhat ABCD véi kich thuée |AB| =20, |BC| = 12. Hinh chit nhat duge chia ra thinh mot luéi 6 vudng g6m 20 x 12 hinh vuong don vi. NI duge phép thye tren mothinh vudng khdc chi néu ich gitta hai tam cia hai hinh vudng ds la /r. Nhiém vu cia chiing ta i én dng tit hinh vudng c6 A 1a dinh ti hinh vudng wéi B (a) Ching minh ring chting ta khong thé thue hién duge nhigm vu trén néu r chia hét cho 2 hoac cho 3, (b) Chimg minh rang nhigm vu tréncé thé thue hign duge néu nhur r 7? (©) Nhi¢m vu trén 6 thuc hign duge khong néu r= 2. Cho P la mot diém nim bén trong tam gidc AABC thoa man ZAPB— ZACB = ZAPC— ZABC. Goi D, B la cée tam dudng tron n mg. Ching minh ring ba dung tip ciia cde tam gife AAPB va APC twong 12 AP, BD vaCE déngquy tai mot 3. Ki higu N 1a tap tit cA ede sO nguyén khong am, Hay xéc dinh tat ca f:N—N thod man F(m+ F(n)) =F(Flm)) +4(n) Vm.neN, Hib) OU HEHEHE 37'h International Mathematical Olympiad Mumbai, India thi thit hai: 9 a.m.= 1:30 p.m say 11 thang 07 nam 1996 ‘Thoi gian lam bai: 45 gi OHI Sree 4. Cée s6 nguyen ducng a vab thoa man cic sO 15a + 16b va 16a — 15b déng thai la cée 86 chinh phuong, Hay x4c dinh gid tri bé nhait 6 thé ciia s6 nhé han trong hai 86 chinh phuongnéi tren ? 5. Cho ABCDEF Wi mot hinh luc gide thoa man AB song song véi DE, BC Kihigu Ra, Ro va Ry tuong ing APAB, ABCD wa alu gidc. Chimg minh ring Rat Rot Rp = why © Ha Duy Hing 38" IMO, tai Mar del Plata, Argentina, nam 1997 61 6. Cho p,q. 5 nguyén dung thoa man p+ q-< 1. Ki higu (179, 71,.++),) 1a mot b6 gémn. + 1 s6 nguyén thoa man cdc diéu kien sau: (a) 20 = 4, =0. (b) Voi méi ima] 1 (b) Chimg minh ring f(m,n) 3 ta c6: 2h < F(a) <2 2.39 Lan thir ba muoi chin, tai Taipei, Taiwan, nam 1998 TT [Ho vaten Lop | Trrng Dim | Huy chuong T 12_[ DHSP Ha Noi 33 HCV 2 [DS Quang Yen | 11 [THPT Lam Son, Thanh Hoa | 29 HCB 3_[ Pham Huy Tong | 12 [DHKATN Ha N 26 HCB [Doan Nhat Duong | 12_[THPT Chuyén ThaiBinh | 24 HB 3_| Dio Thi Thu Ha _|_12_[DHKHTN Ha Noi 3 HCD. 6 [Le Thai Hoing [1 | DHSP Ha Noi 3 HCD Heer OOO VHHEEIHEEHIEE 39! International Mathematical Olympiad Taipei, Taiwan Ngay thi thie nha Ngay 15 thang 07 nam 1998 ‘Thdi gian lam bai: 42 gia Sameer OY OR see SEe 1. (Luxembourg) Trong tit gide I6i ABCD e6 cae dutmg chéo vudng g6¢ véi nhau va cdc canh doi dign AB va CD khong song song véi nhau, Goi P é cia hai dudng trung truc cita cdc doan thing AB va CD va gia sit r trong tt BCD. Ching minh ring ti gide ABCD la mot ta gide noi tigp khi vachi khi cdc tam gide AABP va ACDP cé din tich bang nhau. v (India) Trong mot cude thi ti, 6 «@ thi sinh tham gia va e6 b gidm khio, & 46 b> 3 1A mOts6 Ie. Moi gidm khio dinh gid mdi thi sinh dya trén hai tiéu chudn dé hoae trugt. Gid sit ring k la mot s6 tho’ man, véi bat ki hai gidm Khao ndo, thi cc danh gid cita ho 1A gidng nhau véi it nhat f thé sinh. Ching minh ring ko b=1 a 2b © Ha Duy Hing 39" IMO, tai Taipei, Taiwan, nam 1998 64 3. (Belarus) Véi bay ki sO nguyén duong n, ta xc dinh d{(n) 1a $6 ede use nguyén dutong cia n (ké ca 1 va m). Hay xdc dinh tat cd cdc 6 nguyen dong & dé t6n tai s6 nguyen duong n ma d(n?, dn) Heer OOO VHHEEIHEEHIEE 39! International Mathematical Olympiad Taipei, Taiwan Ngay thi thit hai Ngay 16 thang 07 nam 1998 ‘Thdi gian lam bai: 42 gia beac! OHI 4. (United Kingdom) Hay xii dinh tat c& cc cap s6 nguyen duwong (a, 6) tho man al?-+b-+7 Bi uée cia 0°) +a +b. a tam cita dung trdn ngi tigp tam gide AABC. Dug tron noi tigp tam gid A ABC tigp xtic v6i KL va M. Dutng thing di qua B va song song véi duéng thing MC dutng thing LM va LK lin lugt 6 cdc digm R va S. Chimg minh ring tam gide ARIS 6. Bulgaria) Ki higu Z, I taptit ef ede s6| nguyen duong va xét tat ek ede hdm s6 {:Z, —Z, thoa man f(?/(s)) = s(f(t))? v6i moi s vat thude Z.. Hay xdc dinh gid tri bé nhat c6 thé cia f (1998) 2.40 Lan thit bon muoi, tai Bucharest, Romania, nam 1999 TT [Ho va ten Lop | Trung, Diém | Huy chung Le Thai Hoang | 12 _[DHSP Ha Nor 38 HCV Do Quang Yen | 12_[ THPT Lam Son, Thanh Hoa 36 HCV Bui Manh Hang__|_12_[ DHKATN Ha Noi 29) HCV Pham Trin Quan | 12_[DHKHTN Ha Noi 27 HCB Trin Van Nghia _[12_[ THPT Chuyén Le Khidt, Quang Ngai | 24 HCB fur] s}es) 19] 4} Nguyén Trung Ta | 12_[DHKHTN Ha Noi 23 HCB © HaDuy Hing 40" IMO, tai Bucharest, Romania, nam 1999 65 no Heiser QOD OD eH SEHS 40" International Mathematical Olympiad Bucharest, Romania Ngay thi thie nat Ngay 16 thang 07 ném 1999 ‘Thdi gian lam bai: 42 gia eine rernEE OOO OO Habre ESHR 1. (Luxembourg) Hay x4c dinh tat cd eae tap hitu han c6 it nhat ba diém trong mat phing thod man diéu kign sau: ‘VGi bat ki hai tap con ri nhau A va B nio cia S, thi duémg trung truc cia doan thing AB 18 mot true d6i xing cla S is an axis of symmetry for S. 2. ndia) Cho n IA mot sé nguyen cd dinh vin > 2 (a) Hay xéc dinh hang s6 bé nhiit C tho man bat ding thite ' SY aa(el+e 21a mots6 nguyen duong. Thdi diém ban dau c6 n con bo chét nim trén mot dudng thing nim ngang va khong cing 6 mot diém. Véi méi mot sé thuc duong A, ta xc dinh mot chuyén déng nhusau: Chon bat ki hai con bo chét, tai ca diém A va B, v6i A 6 ben tri ciia B; dura con bo chét & A nhay dén diém C nim trén dung thing =. B 6 va ben phai cita diém B sao cho Hay xi dinh tat c& ede gid tri 06 thé ciia \ sao cho véi moi diém MF nim tren dung thing va bat ki vi tri ban du nao ciia n con bochét, thi e6 mot day hitu han ce chuyén dong sao cho sé dua tat cd cde con bo chét vé ben phai cla diém A. Hee ER OOOO HHH 415¢ International Mathematical Olympiad ‘Taejon, Korean Ngdy thi thie hai: 9 a.m. = 1:30 p.m. Ngdy 20 thing 07 nam 2000 ‘Thoi gian lam bai: 45 gi eine HHE HEE OOO OO Habre 4, Mot nha do thuat c6 100 chiée 1a bai dénh sé tir 1 dén 100. Ong dat ching vao trong ba cai hop, mot hop c6 mau dé, mét hop mau tring va mot hop cé mau xanh, sao cho mdi hop chifa ft nhait mot chiée Id bai. Mot ngudi khén gid chon hai trong, © Ha Duy Himg 424 IMO, tai Washington D.C., United States of America, nam 2001 69 hop ma khong c6 chiée 1é bai no duge chon, Hoi rang c6 bao nhieu céch dé bd tat cd cdc 1d bai vao ba chiée hop sao cho nba aio thuat c6 thé xéc dinh dutgc theo hur tren, (Hai cach duge xem Fa khée nhau néu nhurcé mot [é bai thude vao mot chige hop khéc). 5, Hay xdc dinh xem ligu c6 t6n tai hay khong mot so nguyen duwong n saocho n cd diing 2000 uée nguyén t6 phan bigt va.2" +1 chia hét chon. 6. mottam gide nhon ABC. Duong cfic eanh BC, CA va AB & cic dligm tung ting 1& Ty, Ty va Ts, Xét cdc sinh d6i xting cia cic dug thing HH, HaHa, Va HIT, qua cde duing thang T,T,, T)T, va Ta T. Chiing minh rang cic nhd6 tgo thinh mot tam giéema cée dink nim tren w. 2.42 Lan thit bon muoi hai, tai Washington D.C., United States of America, nam 2001 TT [Ho va ten Lop | Truong Diem | Huy chuong T_[ Va Ngoc Minh T1_[ DHSP Ha Noi 33 HCV 2_[Le Anh Vinh T2_[DHKHTN Ha Noi 28, HCB 3_ [Le Dinh Hing 12 THPT Lam Son, Thanh Hod [26 HCB [Trin Khanh Toan_[_11_[DHSP Ha Nor 23 HCB 5_[Naguyén Anh Quan [12 [NK Tran Pha, Hai Phong [22 HCB 6_| Neuyén Hoang Ding |_12_| THPT Ha Noi-Amsterdam BK Heiser QOD OD eH SEHS 424 International Mathematical Olympiad Washington D.C., United States of America Ngay thi thie nhdi: 9 a.m. + 1:30 p.m. Ney 08 théng 07 nam 2001 ‘Tho gian lam bai: 43 gia Bebe HerHE OOO OO HebHESHnESHR 1. Cho tam giée ABC nhon v6i O 1a tam dug tron ngoai tigp tam gide. Cho PIA ‘mot diém nim trén dudng thing BC va 1a chan cita duéng vudng géc ha tit diém A. Giastr rang 2BCA > ZABC +30°, Chimg minh rang ZC AB + ZCOP < 90°. © Ha Duy Himg 42" IMO, tai Washington D.C., United States of America, nam 2001 70 2. Chimg minh ring 6. a b ey ae SS ee Vet 8be Vi FRca Ve Rab ‘véi moi s6 thye duong a, b vac Hai muoi mot co gai va hai muoi mot chang trai tham gia vao mot eude thi Toan hoe. Diéu thi vi (a) méi thinh vign gidi duge ft nhat sdu bai todn va (b) mdi cap gdm mot trai mot gai thi phai c6 it nhat mot hai todn ma duve gidi ai ca hai ngudi dé, Chiing minh ring e6 mot bai todn duge giai bai trai. ait ba co gai va ft nhat ba ching Hee ER OOOO HHH 424 International Mathematical Olympiad Washington D.C., United States of America Negay thi thit hai: 9 a.m. « 1:30 p.m. Ngdy 09 thing 07 ndm 2001 ‘Thoi gian lam bai: 4 gio Sine rHeHHEE OOOO Habra Cho m 1A mot so nguyén 18 16n hon 1 va choc, Gq Wa cde s6 nguyen. Voi smi hodin vi a= (0,2, ++4, dy) cha tap hop {1 2,++»,m}, ta xdc dinh Sa) = So" Ga. Ching minh rang t6n tai cdc hodn vi va e thod man b c sao cho n! chia hét $(b) — S(c) Ben trong tam giée ABC vé doan thing AP 18 phan gide cia ge BAC, voi P 1a diém nim tren BC, va doan thing BQ la phan gide cia g6e ABC, v6i Q nim tren canh C/A. Gia sit ring ZBAC = 60° varing AB-+ BP = AQ+ QB. Hoi ring cc géc cla tam gic c6 thé bang bao nhieu ? Choa > b> ¢ > d lAcde sO nguyén dong thod min ac+bhd=(b +d +a—0)(b-+d—a+e). Ching minh ring ab +ed khong phai I& mot s6 nguyen t6. © Ha Duy Hing 434 IMO, tai Glasgow, Scoland, United Kingdom, nam 2002 on 2.43 Lan thit bon muoi ba, tai Glasgow, Scoland, United Kingdom, nam 2002 TT [Ho va ten Lop | Trutmg Diem | Huy chuong T_|Pham Gia Vinh Anh | 12_[DHSP Ha Noi 35 HCV 2_[Va Ngoc Minh 12_[ DHSP Ha Noi 35 HCV 3_[Nguyén Xuan Trong__| 12_[THPT Chuyen Vinh Phas | 29) HCV 4_| Pham Hong Viet 12_[ DHKATN Ha Noi 24 HCB 5_| Pham Thai Khanh Hep |_11_[ DHSP Vinh 22, HCD 6 [Mai Thanh Hoang. 12_[DHSP Vinh 2I HCD 26.2728 A IMO 200 eloran Hinh 2.2: Logo cita IMO 2002 tai Glasgow, Scotland, United Kingdom. 43° International Mathematical Olympiad Glasgow, Scoland, United Kingdom igdy thi thit nhit gay 24 thing 07 ndim 2002 hoi gian lam bai: 44 giar amg THPT Chuyen Dai hoc Su pham Ha Noi ki thi IMO va la hocsinh thi tu ca Viet Nam 6 duge én Khoi THPT Chuyén cia trutng 1g trong mot ki thi IMO. n Truong [a hoc sinh dau tién cua tinh Vinh phic duge tham dy mot ki thi IMO, (gc sinh du tién cita tinh Vinh Phiic doat huy chuiong Vang tai IMO. © Ha Duy Himg 43" IMO, tai Glasgow, Scoland, United Kingdom, nam 2002 nD v . Cho n La mot so nguyen duong. Ki higu T fa tip tat ed cdc diém 6 toa d (x,y) trong mat phing & dé x va y 1a cdc sO nguyén khong am va r+ y 3 thod min diéu kig han s6 nguyen dong a sao cho a™+a-1 1a mot s6 nguyén. Heiser QOD OD eH SEHS 434 International Mathematical Olympiad Glasgow, Scoland, United Kingdom Ngay thi thie nha Ney 25 thang 07 nam 2002 ‘Thoi gian lam bai: 42 gio eine rHcrEEE OOO OO HebHESHRESH Cho n 1A mot s6 nguyen Ién hon 1. Céc ude nguyén duong eita n duge ki higu dy .day2025dy 646 1 = dy < dy < +++ < dy =n. Xée dinh D = dydy+ dody+ ve tdy ids (a) Ching minh ring D< (b) Hay xéc dinh tat cd cdc sé. n ma D Ia uée cian” . Hay xdc dinh tit ed céc him f sR —+ R sao cho (Fo) + F(2) Fu) + FQ) = Fey 2t) + flat +2) véimoic.y, 2.0 R. © Ha Duy Hing 434 IMO, tai Glasgow, Scoland, United Kingdom, nam 2002 B VES 6. Chol, Py,.+4,Ty Hede dudng tron bin kinh 1 trong mat phing, & 46 7 ki higu cdc tm ciia né tuong ting Ia O}, Oo, «++, On. Git sit ring khong 06 duéng thing no cit qua hai duémg tron trong 86 ce dudmg tron néi tren. Prove that TT [Ho vaten Lop | Trutmg, ém | Huy chuong T [Le Hing Visto | 11_| DAKHTN HG Nor a HCV 2_[Neuyén Trong Canh | 12 | DHSP Ha Noi a HCV 3_[Nguyén Dang Khoa | 1_|DHKATNTPACM 26 HCB 4_[Neuyén Ding Hop | 11_[ THPT Le Hong Phong, Nam Dinh | 23, HCB 35_[Neuyén Tign Viet__[ 11_| THPT Le Quy Don, Khanh Hoa [21 HCB © [Vi Nhat Huy 12 [ THPT Chuyén Vinh Phic 18 HCD 29 30 > Nguy! (0¢ Sinh thi hai d&n tir Khoi THPT Chuyén trugng DHSP Ha Noi dat s6 diém t6i da trong mot ki thi IMO. im 2003 1 nam di tien Viet Nam c6 hai hoe sinh ci t6i da tai IMO. doat huy chuong Vang véi sé digm © Ha Duy Hing 44!" IMO, tai Tokyo, Japan, nam 2003 4 Hinh 2.3: Logo cita IMO 2003 tai Tokyo, Japan, Hee ER OOOO HHH 44th Thternational Mathematical Olympiad Tokyo, Japan Ngdy thi thie nhdi: 9 a.m. = 1:30 p.m. Ngay 13 thing 07 nam 2003 ‘Thoi gian lam bai: 45 gi eine HHE HEE OOO OO Habre 1. (Brazil)Cho.Al& mot t4pcon etia tap hgp {1,2,3,..., 1000000} 86 ti, toyeeey tion trong S sao ={rtt|reA} v6 +100 1a doi mot khong giao nhau. v (Bulgaria) Hay xéc dinh tat cd cic cAp s6 nguyén duong(a,)) thod min a TPP T i mot s6 nguyén dong 3. (Poland) Mot luc gic I6i thoa man véi bat ki hai canh doi dign no cia né thi khodng cich gitta cdc trung diém cia chting bing V'3/2 Kin tong d0 dai cia cic canh, Ching minh ring tat c4 cc géc cita luc gide Ia bang nhau. (Mot luc gide 16i ABC DEF c6 hai cap canh doi dign dé Ia: AB va DE, BC va EF,CDWPA) © Ha Duy Himg 44! IMO, tai Tokyo, Japan, nam 2003 75 Sem OY 44! International Mathematical Olympiad Tokyo, Japan Ney thi thit hai: 9 a.m, = 1:30 p.m. Ney 13 théng 07 nam 2003 ‘Thdi gian lam bai: 42 gia Heer OOOO HHEEHEEHIEE 4, inland) Cho ABCD 1a motte gi cdc dutng vudng géc duge ha tir D xudng eae dung thing BC,CA ting. Chg minh ring PQ =QR khi va chi khi céc dung phan gi ABC va ADC cat nhau tai mot diém nam trén doan thing AC. 3! 161 tiép . Cae diém P,Q va R Ia chan ciia 1B, twong clia ede géc 5. (Ireland) Cho nla 6 thye thed man TSB SeS nguyen duiong va 21,2. (a) Chimg minh ring (b) Chimg minh rang bat dang thite xdy ra dau bang khi va chix), 29, thanh mot cap s6 cong. &, lap 6. (France) Cho p1A mots6 nguyen t6. Ching minh ring tén tai mot s6 nguyen t6 4 thod man doi véi moi so nguyén n, thi s6 n” — pkhOngchia hét cho q. ic ABCD noi tigp fa khong céin thie "Chi thich boi ngudi dich: Tren thu té diéu kien tt gi © HaDuy Hing 45" IMO, tai Athens, Greece, nam 2004 16 2.45 Lan thit bon muoi nam, tai Athens, Greece, nam 2004 TT [Ho va ten Lop | Trutmg Diém | Huy chuong T_[Pham Kim Hung T_[DHKHTN HaNoi 37 HCV 2_[Lé Hing Viet Bio | 12_| DHKHTN Ha Noi 36 HCV 3_[ Nguyén Kim Son 12 35 HCV 4_[Neuyén Minh Trung | 12 35 HCV 35_[Neuyén Die Thinh__|_11_[ DHS 27 HCB © [Hita Khic Nam 12 [DHSP Hi Noi 26 HCB nue Heer OOO VHHEEIHEEHIEE 45! International Mathematical Olympiad Athens, Greece Ngdy thi thie nhdi: 9 a.m. - 1:30 p.m. Ngdy 12 thing 07 ndm 2004 ‘Thoi gian lam bai: 4 gio ee EA OOOO HHH aT RAO) RT SRS ineolntariney HELLAS 2004 Hinh 2.4: Logo clia IMO 2004 tai Athens, Greece. =Day Ia Hin ddu tien Viet Nam dinh dugc bon huy chuong Vang Le Hing Viet Bio tro thanh hoe sinh th nam ciia Viet Nam hai lin doat huy chuong Vang tai IMO. Tong s6 diém cita Bao dat duc qua hai Ian thi 1a hien dang ding 6 vi tri tii tu so véi céc hoe sinh Viet Nam daitimg tham dur IMO, tinh tir trude dén nay. “!Nguyén Minh Truong 13 hoe sinh thi hai ciia NK Trdn Phi, Hai Phong doat huy chuong Vang tai IMO. Hign nay ‘Trung van la hoc sinh ciia Hai Phong dat s6 digm cao nhalt trong so ac ban da tham gia IMO. mot ki thi IMO, © Ha Duy Hing 45!" IMO, tai Athens, Greece, nam 2004 1 1. Cho tam giée AABC nhon véi AB 4 AC. Dutng tron dudng kinh BC lin luot ct cde canh AB va AC & cae diém M_ 7 va N. Kihigu O 1a tung diém cia canh chung nim trén canh BC. 2. Hay xéc dinh tit cd cdc da thie / véi cdc hé s6 thyc tho min véi moi a,b, cthye maab + be +ca=0 thi ta 6 fla—b)+ f(b—e)+ flea) =2f(a +b +e) 3. Ta xéc dinh mot edi fugi cau 1 mot hinh duge tgo boi su hinh vudng don vi chi ra é hinh ve dudi day, hoge Ia bat ki hin no e6 thé nhan dutge tit hin dé qua cdc phép quay va d6i xing vao hinh nay. Ta xét tit cd cdc hinh chit nhat kich thude m x n ma cé thé phi duge béi cdc Ludi edu kigu trén sao cho déng thdi thoa man hai diéu kién sau day: (a) Hinh chit nhat duge phi khong ¢6 16 thing nao va cting khong ¢6 hai lua cau no chong len nhau, (b) Khong phiin nio cia mot udi cau phi phan ben ngoai cia hinh chit nat. inoliftanianr Pr HELLAS 004 [ebibicereeee DOOD HHHEEEEREEE 45¢h International Mathematical Olympiad Athens, Greece Ngdy thi thit hai: 9 a.m, = 1:30 p.m. Ngdy 13 thang 07 nam 2004 ‘Thoi gian lam bai:43 gid [ebrbrcbereeee DOOD HEE © HaDuy Hing 45" IMO, tai Athens, Greece, nam 2004 78 4. Chon > 31a mot so nguyen. Kf higu f;,%»,.-., fy IA cde so thuc dong thod man ars) Ching minh ring véi moi b@ ba chi s6 i, j, kthod man] (tht: ZPBC=ZDBA & ZPDC=ZBDA Ching minh rang ABCD la mot té giée noi tiép khi va chi khi AP=CP. 6. Tandi mot so nguyen duong Ia thay phién néu v6i bat ki hai chit so lien tigp no trong biéu dig 6 he thap phan cia né déu cling khéc nhau tinh chin 18. Hay xic dinh tat ca cdc sO nguyén dong n thoa man n ¢6 mot boi nguyen I& shay phién. Compiled by Ha Duy Hung Email: pluricomplex_amath@yahoo.com Url: www.mathseope.com Hanoi University of Education Vietnam. Chuong 3 soe Ldi giai cc bai thi Olympic Toan Quéc té 3.1 Lanthirbén muoi nam, tai Athens, Greece, nam 2004 1 |. Ta goi LI ching minh hi cam on dén nguisi ban rat than Babis Stergiou, Chalkida, ing xin cam on TS, Kin Yin IMO 2004. ' Trong phan nay t Greece da cung cap cho t6i tir rat sém IMO Shortlisted 2004. Toi Li, Hong Kong da rat nhigt tinh gifi cho toi dap in chinh thite 19 Ha Duy Hung Loi gid céc bai thi Olympic Todn quo té 80 Tacé (XM.X XM,XL)=(BA,BC)=(NM,NA) (mod 180°) Hay bon diém X,.M, A, N cing nim trén mot duéng tron, Tir dé ta lai cd (MN, MX) = (AN, AX) =(AX,AM)=(NX,NM) (mod 180°) He thie nay ching t8 qua phép di xsing true a tung true cia doan thing, MUN FI sl ig MN. Vay hai dudng tron (BR) va (CNR) cat nhau tai hai diém 1a R va nhau trén dudmg thing BC. © diém A thinh chinh n6. Phép HT bién trung diém cia doan thang BC thanh trung digm cia doan thing MIN hay néi cach khic bign diém O thinh diém P. TAP (vi AR bién thinh chinh Digu nay chiing t6 rang AR I& phan gic cita g6e O n6) hayta cé AO _ RO >= Ter me AO_ BC _MO ~ MN” MT RT MT hay MRIA phan gidc cia g6e VIVO. Vi ZN MO = ZA nen tit giée AM RN ndi tigp, thank thir giao diém thet hai ca hai dudng tron (BAIR) va (CNR) nam tren Ha Duy Hung Loi gid céc bai thi Olympic Todn quo té 81 duéng thing BC va thang hang véi 2, A tite chinh la diém L. Nhan xét, Trong lai gidi 2, ta c6 sir dung mot két qua quen thude sau tam gide AABC cic diém M va N tuy y trén cae duéng than mg. Ching minh rang néu c6 diém F tho’ man céc tit digm { {A.M R,.N} d6ng vien thi hai duong tron (BR) va (CN R) eat nhau tai thi hai nim trén dudng thing BC. Ching minh, Goi I 18 giao digm thit hai ca hai dudng tron (BIR) va (CN R) Khi dé ta 6 (LB, LR) =(MB, MR)=(NA,NR)=(LC,LR) (mod 180°) Hay LB = LC tite 1a ba diém L, B,C thing hing. © Qua nhan xét néi trén ta thay rang bai todn vin ding néu nhu ta thay duéng trdn duimg kinh BC boi dutmg trdn tam O di qua hai diém B,C 1a dit 6 2. True het ta thay P(x) = aa + be? Rk mot nghigm eda phuong tinh him. Ta chi ra tat c& céc da thite P(r) thoa man phuong trinh him trong bai déu phai c6 dang tren. Lai gidi 1. Voi moi x © B ta c6 6 ba (a, b,¢) = (Gir, 30, be + ea = 0. Thay thé vao phuong trinh ta nhan duge két qua la thoa man ab + P(3e) + P(5x)+ P(-8x) = 2P(7x) Ta viet P(a:) = S7f_g axe* thi ta nhan duoc két qua la (35 +58 + (-8)*§-2-7*)aq =0 v6imoi k =0, 1,044, 7. Xét phuong tinh 3k45h+ (-8)*-2-7§=0, ‘v6i din nguyén khong am k. Phuong trinh nay chi cé hai nghigm Ia k = 2 hode 1a k= 4. Nhu vay, v6i moi k #2, 4 thi ay = 0. tir d6 ma P(r) = ax! + br? véi a, b tye no d6. Thit lai dé thay 46 nghiem ciia phuong trinh ham, © Lai giai 2. Trude hét ta xét bd ba (a,b,c) = (0,0, ¢) hién nhién thod man ding thtfe ab + be +ca = 0. Ta nhan duge P(0) + P(e) + P(e) = 2P(c) hay P(e) = P(0) + P(-c) vi moi c thye. Dac bigt thay ¢ = 0 ta nhan duge P(0) = 0. Ha Duy Hung Loi gid céc bai thi Olympic Todn quo té 82 Thanh thir P(c) = P(—c) véi moi c € R. Tir dé tén tai mot da thite Q(x) ma P(e) = Q(x”). Theo gia thidt ta c6 SY Q(a=6)?) =2Q((a+b+e)") Va,bceR eyelic Dang phép déi bign ding thitc ab + be + =e sym eychie 2-3! v6i moi i > 3 (diéu nay 6 thé suy ra tit bat dng thie Bernoulli (1 +a)" > 1 +na véi moi a > 0,n € Z,). Mat khéc vi P(0) = 0 né Q(0) = 0. Tém lai Q(x) = ax? + be tit dé ma P(r) = ax! +b: Lai giai 3. Trude hét ta ciing lam nhu trén dé suy ra P(w) = P(—x) véi moi aw R. Ta viet -2 ' P(e) = ayn" + gy 98"” shage? + ay Ha Duy Hung Loi gid céc bai thi Olympic Todn quo té 83 Voi miu, v € B ta xét bo ba (a,b,c) = (ww, (1 — uv, (02 — ww) dé thay n6 thoa min dang thtic ab + be + ea = 0. Thay vao phurong trinh ham ban dau ta nhan durge ket qua la P((Qu=1)v) +P((1—02)v) + Pl(u? —2u)v) =2P((u2—ut Iv) YuveR Bay gid v6i mdi 1 ta xem céc biéu thie tren nhurnhiing da theée clta v déng nhat cache s6 ta duge (Qu 1)?" + (1 = 2)? + (0? = 2)" = Au? a1)" VueR Lay u 2 ta nhan duge 57" +37" + 8°" = 2-7". Tir day dé y rang 8" > 2-7" véimoi n> 3. Thanh thir n < 2, lai cht ¥ rang P(0) = 0 nén ta nhan duge dang cintim. © (Loi gidi chinh thiée). Xét mot phii cia mot hinh chit nhat kich thuée m x n thoa man hai yéu cdu cia bai todn. Véi bat ki mot lui cau A no, déu tdn tai duy nhat mot lui cau B phit mot trong céc hinh yudng nam bén trong A va nim tan cing hur vay, tat eae Iu6i cau sé durge sp xép khép nhau thinh cdc cap. Chi cé hai kha nang c6 thé dat ludi ctu B sao cho né khong chéng lén A va khong c6diém thing nao. Trong hoan cinh thie nhit, A va B tao ra mot hinh chit nhait 3 x 4; cdn trong trudmg hop cdn lai hgp cila chting la mot hinh bat gidc, véi d6 dai cdc canh 14 3, 2,1, 2.3, 2,1, 2 (ta tam goi lA céc hinb lat). Do vay mot hinh chit nhat kich thude Iam xn e6 thé durge phii boi ede luGi cau Khi vAchi khi n6c6 thé duge phi béi cc hinh Lét gm 12 hinh vuong & dang nh néi 6 tren. Mot céch phit nhut vay t6n tai thi mn phai chia hét cho 12. Bay gid’ ching ta sé ching minh rang trén thuc t€ mot trong hai s6 m va n phai chia hét cho 4. ‘Thue vay, phan ching ring diéu dé khong xay ra; khi d6 m,n I& cdc sO chin va khong chia hét cho 4. Chiing ta hay hinh dung ring hinh chit nhat duge chia thanh cc hinh vudng don vi, ta dénh s6 cdc hang, cOt 1 1,...,m va 1, 2,...,n. Ta viet 8611 vio hinh vuong 6 toa dé (i, 7) néu cé ding mot trong cdc 86 ihoae j chia hét celia A (mdi Ludi cau cé bon hinh vudng nhu thé). cho 4, va viet vo s62 néunhur ca j va j cing chia hét cho 4. Virang s6 ede hinh ot la chain, nén téng cac sé di duoc viet at phai tra thay ring hinh chit nhat 3 x 4 Iu6n ludn 6 thé phii cdc so c6 téng bang 3 hoac 1&7; trong khi dé cdc Lat 12 inh vudng khéc c6 tong cai 5 hoac la 7, Hé qua ciia diéu nay la tong tat ca cae s6 cita cae lat 12 hinh vuong 1a chin, Nhung khi dé mn chia hét cho 24 vad6 1a diéu khong thé. Chi ¥ rang cde so m va n khong thé déng thai bang 1, 2 hoac bang 5. Nhu vay chiing ta da vita chi ra ring néu mot phi c6 thé duge thuc hign thi m,n trong hai s6 c6 mot s6 chia hét cho 4,3 vam.n ¢ {1,2,5} Ta sé ching minh ring tat ca cde cap s6 nguyen duong (m,n) vita ligt ké thoa man yeu céu cita bai toan. Thue vay, ta chi cin sit dung ca lat Ia cdc hinh chit nhat c6 kich thude 3 4. Diéu nay hién nhién thyc hién duoe khi ma 3 18 ude cla medn 4 1a we ciia n, Néu m Ia s6 chia hét cho 12 van ¢ {1,2,5}, thi n c6 thé biéu dign vwuong theo cdc ding va theo 1 86 chin, Bay gids dé dng ki Ha Duy Hung Loi gid céc bai thi Olympic Todn quo té 84 duge nhu 1a téng cia mot vai s6 3 va mot vai sé 4. Do dé ma hinh chit nhat mx n €6 thé duge phi bai céc hinh chit nhat dang m 3 vim X 4. Hai loai hinh chit nat dé lai dé dang duge phi bai cdc hinh lat 3 X 4 va do dé bai tosin duge gidi quyét hoan toan. 6 chtmg minh khang dinh ciia bai toan bang phép quy nap todn hoc. Trude het hay xét trudc hop n= 3, gid sit ring chiing ta 66 ba s6 duong a,b, ¢ thoa andin bat dng thie 1 10> (a+b-+e)+ (@ + ‘) G3) be Khong gidm tinh tng quat, gid sit ring a 1a sO 1én nha trong ba s6'a, b,c. Khi dd hign nhien ta c6 cdc bat dang thite a +b > ¢ vaa + > b, Nhu vay, van dé cdn lai chi la ching minh bat ding thite a (a+b+e)+ (+h+)>e ‘Ot tinh toan dai s aabtee orbea-(S+5 Thay vio (3.4) ta nhan duge ket qua fa =) GA) don gidn cho ta to>54+ 4 hay 4x? — 5e-+1 < Otrong dé Néur > 1 thi de? 52r+1= Lo(e= 1) + (1-2) =(e=1)(4e- 1) ea do d6 mau thu. Vay ax < Thay a 3. Xét n +1 s6 duong ty, to, 004, tn 4) thod man bat ding thite di cho trong trugng hop n+ 1. Ta can ching minh basé khéc nhau doi mot bat Ki déu e6 thé ly lam ba canh ciia mot tam gice, Néu ciin danh lai céc chi s6 ta c6 thé xem ba s6 cn chi ra thoa man cic bat dang thc tam gidc thude vao tap cic $6 fp, fy,-++5 ft Tacs (n+ 1P 41> (t+ tote +t 5 1 Sl+t (2 Ha Duy Hung Loi gid céc bai thi Olympic Todn quo té 85 646 1 1 (ty teeet ty oe(geet 4) Theo bit ding thie Cauchy Tir(3.5) tacé (n+1P4+1>14h- Sl+In+e Thanh thir A 3. © Lai gidi 2. Khong gidm tinh téng quat ta chi cén ching minh f ( ia n? 44404 > de? Sr +1 <0 1 Ho o t+ Ly thi TN? teu? ety -(B-VE) (w=)? | (a=) he tits tite 5 a= bt, (te = Ht The tite hi G_ bith a ee Paty teotst © dinh duge s6 f(n) i, thod man bat dang thie Nhan xét. Tic gii ciia bai todn dy thong bio ring c6 t 16n nhat sao cho bat kin s6 duong ¢),.. Yue dees ket mi thi ba s@ bat ki trong chiing ¢6 thé lap 1am ba canh ciia mot tam gic. Thue t€ bing ‘mot lap luan tuong tr nhu 16i giai 2 ¢6 thé chig minh duge f(n (nevi 5. Goi K va lL va DP. Ta (BP, Bi thité gi giao diém ciia dudng thing AC tuong tng véi cdc dudng thing BP ching minh khang dinh manh hon, néu P 1a mot diém thoa man, (BA, BD) (mod 180°) va (DP, DC) = (DA, DB) (mod 180°) BCD noitigp khiva chi khi AP =CP. 22, we Truéc hét ta gid sit ring tit gidc ABCD Ia mét tit gide noi tiép. Xét hai tam gidc AADB va AKCB, theo gid thigt (BA, BD) = (BK, BC) (mod 180°). Do bon diém A, B,C, D cing nim trén mot dudng trdn nen (DA, DB) = (CK,CB) (mod 180°). Vay AADB~ AKCB, suy ra DB_AD , , AD_CK BD CB (3.6) Ha Duy Hung Loi gid céc bai thi Olympic Todn quo té 87 Bay gid xét hai tam gidc AAD va ABDC, mot lin nita do bén diém A, B, cing nam trén duéng tron nén (AD, AC) = (BD, BC) (mod 180°). Mat khéc, tir gid thiét (DA, DB) = (DP, DC) (mod 180°) va hé thie Chales cho phép ta nhan duge (DA, DL) =(DA, DB) + (DB, DP) (mod 180°) = (DP, DC)+ (DB, DP) (mod 180°) =(DB,DC) (mod 180°) Vay tacé AADL ~ ABDC, suy ra ring AD _ AL BD BC GB.7) ta nhan duge ket qua la CK = AL. Phép di xting true, véi ia doan thing AC bién diém A thinh diém C, do dé dé ching ‘hig minh diém P bién thinh chinh n6, Thye vay theo he thtic Chales (DP. AC) = (DP, DC) + (DC, AC) (mod 180°) = (CA,CB)+ (DC, AC) (mod 180°) DC. BC) (mod 180°) va tit nhién, tuong ty, tacting c6 (BP, AC) =(BP, BC) +(BC, AC) ) = (BA, BD) +(BC, AC) (mod 180°) CA,CD) + (BC, A (mod 180°) ) =(BC,CD) (mod 180° (mod 180° Ha Duy Hung Loi gid céc bai thi Olympic Todn quo té 88 Két hop hai dang thic, ta duge (BP, Al (DP, AC) (mod 180°). Nhu vay qua phép doi xiing truc dang xét dudng thing BP bien thinh dudng thing DP. Vay diém P bién thinh chinh né, hay néi céch khdée P nam trén trung truc cha doan thing AC. Do vay PA = PC. Bay gid ta gid sitrang AP bang vai CP. Ta goi X,Y Vin lugt 1a giao diém thet hai cia dutmg tron ngoai tigp tam gide AB PC véi cac dudng thang DC va DP. Xét hai tam gidc 18 ADAB va tam giéc ADPX, theo gid thigt (DA, DB) = (DP. DX) (mod 180°). Lai c6 bon diém B, P,C, X déng vien nen, (XP, XC) =(BP, BC) =(BA, BD) (mod 180°) Vay ADPX ~ ADAB, do dé ma Mot mat (DA, DP) = (DB, DC) (mod 180°) nén ta nhan duge ADAP ~ ADBX. Tit day nhan duge DA P DB BX Suy ra DP DX Ha Duy Hung Loi gid céc bai thi Olympic Todn quo té 89 Do AP = CP nén suy ra XY = BN haitam giée BXY can tai B. Bay gid dé ket thie chimg minh can mot vai tinh todn don gin ngin dudi day (CD, CB) = (YX,YB)=~(BX, BY) =—(PX,PY) (mod 180°) DP, XD) +(XD,XP)= (DA, DB)+(BC, BP) (mod 180°) =(DA,DB)+(BD, BA) (mod 180°) (DA, BA) (mod 180°) Vay tit diém A, B,C, D déng vien va bai ton duge ching minh hodn toan. Nhan xét. Trong Idi giai 1, diéu kien cén, nghia 1a te g taciin rit ra khang dinh AP dep hon nhut dud day: ABCD Wa noi tigp va ve nhe nhaing hon va Goi Uva V tuong ting Ia cdc giao diém thit hai ciia cdc dung thing BP va DP v6i dudng tronngoai tigp tt gide ABCD. DP OBP={P}nén P nam trén truc déi ximg, nghia la P céch déu hai digm A va. © Ha Duy Hung Lai gi cdc bai thi Olympic Todn quée 16 90 hai chi s6 cuGi cling déu Ia cae s6 chain do dé khong 6 mot boi nguyen thay phién no ca. Bé dé 1. MGi lus thira cia 2 c6 mot s6 boi thay phien. Ching minh, Ta sé xay dung mot day vo han {a,,} 5 ede chit s6 tha man a Sn+1 (mod2); 2° la;ce as Pama TT véi moi n nguyén duong. Thue vay diu tién lay a, = 2, ay = 7 va gia sit ring ta da xay dung duge dén ayn 18 86 1é. Lay ay,..1 = 4, khi dé ta cé, 42 Tai = 4-10" +O -5°" 4 Opie do d6.2°"||mrit-- TH. Bay gid tacdin tim ra chit so 18 az, saocho || TT ‘Theo gid thiet quy nap, ta 06 thé viet Taya +h trong dé hla mot 86 nguyén duong 1é. Ta c6 Tapa == = Gans g 102+! 4 22"). h chia het cho 2?"+5 ya khong chia hét cho 22"*# khi va chi khi G8) = 1 (mod 8), thinh thir 6 dang tuong duong Hay chai ¥ ring v6i moi sO nguyen duong n thi 5°* 5+! = 5 (mod 8) nén diéu kign (3.8) viet don gidn I 4 | 5rOanio th 4 | Ganja — 3h os 3.9) {ire +h {ijn 3h 6%) MOt chit s6 azq..2 thod min (3.9) la t6n tai, trén thue 18 tachi cn cho a2, ,1 chay qua mot he thang du day dii modulo 8, chang han azp..1 chay qua {0,1,2,+++,7} B6 dé duge ching minh hodn toan. © Bé dé 2. Mai s6'c6 dang 2-5” déu cé mot boi nguyén Ia mot s6 thay pl Ching minh, Ta xay dumg bang quy nap mot day vo han céc chit s6 {b,,}525 thoa man bp =n+1 (mod2), 5"||Dre Dr Dau tién ta chon b: 5 va gid sir ring ta da xay dumg duge dén b,. Ta SD, 20) = 5" {4 mOt SO nguyén dung khong chia hét cho 5. Ta cin chi ra su tén taicla mot chitsé 6,.,; thod man bay M42 (mod 2), 5"*"||Fnan Ha Duy Hung Loi gid céc bai thi Olympic Todn quo té 1 Bing cach viet Duy Dye--1 = bpp: + 10" +5" + 2 thi digu kien cudi cing twong duong véi tng =n +2 (mod 2) Bllbnet 2" +2 ‘Trude het chiing ta thay ring quan hé thtt hai cho ta it nhat 3 nghiém theo modulo 25, chang han nhimg nghiém ma b,.,+2"+ c= 5, 10,15 (mod 25). Trong ba sé nay At c6 hai sé khdc tinh chan 1é. Trong hai s6 dé at phai c6 mot s6 Lam thoa man quan hé dau tién, thinh thir hé néi trén ludn co nghiém. Néu ciin ta lly s6 dr cla a1 cho 10, bé dé 2 duge ching minh hoan toan. © Cudi cing ta céin them mot két qua phy try quen thude sau Bé dé3.? Cho A, k lacdc s6 nguyén duong trong dé & nguyén t6 cling nhau véi 10. Khi dé luon t6n tai mot bOi nguyén duong cia k 6 dang AA. A, trong dé n naa a mot sO nguyen duong nao ds, TA,..., TA trong k-+1 so nay khi dem EH chia cho h &t ¢6 hai s6 c6 cling s6 du. Gia sit ring hai s6 46 Ching minh, Ta xét dy cdc 6 A, dA {va dA A ade AW AAA - tA GA vil $j

Potrebbero piacerti anche